NM Update Course II: Inflammatory /Myotonic Disorders

Richard J. Barohn, MD Steven A. Greenberg, MD Bassam A. Bassam, MD Bakri H. Elsheikh, MBBS, MRCP Annabel K. Wang, MD Gregory T. Carter, MD, MS Andrew W. Tarulli, MD

AANEM 59th Annual Meeting Orlando, Florida

Copyright © September 2012 American Association of Neuromuscular & Electrodiagnostic Medicine 2621 Superior Drive NW Rochester, MN 55901

Printed by Johnson Printing Company, Inc. 1 Please be aware that some of the medical devices or pharmaceuticals discussed in this handout may not be cleared by the FDA or cleared by the FDA for the specific use described by the authors and are “off-label” (i.e., a use not described on the product’s label). “Off-label” devices or pharmaceuticals may be used if, in the judgment of the treating physician, such use is medically indicated to treat a patient’s condition. Information regarding the FDA clearance status of a particular device or pharmaceutical may be obtained by reading the product’s package labeling, by contacting a sales representative or legal counsel of the manufacturer of the device or pharmaceutical, or by contacting the FDA at 1-800-638-2041.

2 NM Update Course II: Inflammatory Myopathies/Myotonic Disorders

Table of Contents

Course Committees & Course Objectives 4

Faculty 5

Therapy for Inflammatory Myopathies 7 Richard J. Barohn, MD

Inflammatory Myopathies 15 Steven A. Greenberg, MD

Myotonic Dystrophies 21 Bassam A. Bassam, MD

Nondystrophic Myotonic Disorders 27 Bakri Elsheikh, MBBS

Neuromuscular Vignettes 33 Annabel K. Wang, MD Gregory T Carter, MD, MS Andrew W. Tarulli, MD

No one involved in the planning of this CME activity had any relevant financial relationships to disclose. Dr. Barohn is involved in Speaker’s Bureaus with Genzyme and Grifols and is on Advisory Boards for both MedImmune and Novartis. Any conflict of interest has been resolved according to ACCME standards. Dr. Greenberg is on the advisory boards for MedImmune and Novartis and has sponsored research from MedImmune. Any conflict of interest has been resolved according to ACCME standards.

Chair: Dianna Quan, MD

The ideas and opinions expressed in this publication are solely those of the specific authors and do not necessarily represent those of the AANEM.

3 Objectives

Objectives - Participants will acquire skills to (1) utilize a pattern recognition approach elucidated through clinical vignettes in the diagnosis and management of patients with inflammatory myopathies and myotonic disorders, and (2) practice the vignette-based format used for many questions on the NM medicine board examination. Target Audience: • Neurologists, physical medicine and rehabilitation and other physicians interested in neuromuscular and electrodiagnostic medicine • Health care professionals involved in the management of patients with neuromuscular diseases • Researchers who are actively involved in the neuromuscular and/or electrodiagnostic research Accreditation Statement - The AANEM is accredited by the Accreditation Council for Continuing Medical Education to provide continuing medical education (CME) for physicians. CME Credit - The AANEM designates this live activity for a maximum of 3.25 AMA PRA Category 1 CreditsTM. If purchased, the AANEM designates this enduring material for a maximum of 5.75 AMA PRA Category 1 CreditsTM. This educational event is approved as an Accredited Group Learning Activity under Section 1 of the Framework of Continuing Professional Development (CPD) options for the Maintenance of Certification Program of the Royal College of Physicians and Surgeons of Canada. Physicians should claim only the credit commensurate with the extent of their participation in the activity. CME for this course is available 10/2012 - 10/2015. CEUs Credit - The AANEM has designated this live activity for a maximum of 3.25 AANEM CEUs. If purchased, the AANEM designates this enduring material for a maximum of 5.75 CEUs.

2011-2012 Course Committee

Shawn J. Bird, MD, Chair Shashi B. Kumar, MD Marcy C. Schlinger, DO Philadelphia, PA Tacoma, WA Bath, MI

Lawrence W. Frank, MD A. Arturo Leis, MD Nizar Souayah, MD Elmhurst, IL Jackson, MS Westfield, NJ

Taylor B. Harrison, MD Benjamin S. Warfel, II, MD Atlanta, GA Lancaster, PA

2011-2012 AANEM President

John C. Kincaid, MD Indianapolis, IN

4 NM Update Course II: Inflammatory Myopathies/Myotonic Disorders Faculty

Richard Barohn, MD Bassam A. Bassam, MD Department of Neurology Neuromuscular Program and EMG Laboratory University of Kansas Medical Center University of South Alabama Kansas City, Kansas Mobile, Alabama

Dr. Richard Barohn is chair of the Department of Neurology at Dr. Bassam completed residency training in neurology and a the University of Kansas Medical Center (KUMC). He received fellowship in neuromuscular disease at Wayne State University, his medical degree from the University of Missouri-Kansas with additional fellowship training at Mayo Clinic. He is board City School of Medicine and completed a neurology residency certified by the American Board of Psychiatry and Neurology and at Wilford Hall United States Air Force Medical Center and a the American Board of Electrodiagnostic Medicine (ABEM) and neuromuscular fellowship at Ohio State University. Before joining is a diplomate in the neuromuscular medicine sub-specialty. Dr. KUMC, he developed and directed the clinical neurophysiology Bassam has served on various AANEM committees, including training program at the University of Texas Southwestern. Dr. chair of the Workshop Committee; he current serves on the ABEM Barohn is the author of more than 160 journal publications, 260 Examination Committee. His academic interests and achievements abstracts, and 40 book chapters on neuromuscular disease. He has focus on neuromuscular disorders and . served on the editorial boards for Neurology, Muscle & Nerve and the Journal of Clinical Neuromuscular Disorders. He currently Bakri Elsheikh, MBBS, FRCP (Edin) serves on the Guillain-Barré Syndrome Foundation International Neuromuscular Division and has served on the medical advisory boards for the Myasthenia Ohio State University Gravis Foundation of America and The Myositis Association. Columbus, Ohio He also serves on the executive committee of the national ALS Research Group. Dr. Elsheikh is associate professor of clinical neurology at The Ohio State University (OSU) Wexner Medical Center, where he Steven A Greenberg, MD serves as the director of the EMG laboratory, co-director of the Department of Neurology Myasthenia Gravis Clinic and education program coordinator Harvard Medical School for the neuromuscular medicine and clinical neurophysiology Boston, Massachusetts (electromyography emphasis) fellowships. He received his medical degree from the University of Khartoum, Sudan and his Dr. Greenberg is a clinical neurologist specializing in postgraduate internal medicine degree from the Royal College neuromuscular disorders. He completed his medical degree of Physicians in the United Kingdom. His neurology residency at Harvard Medical School and a neurology residency and and neurophysiology/neuromuscular fellowship was completed neuromuscular disease fellowship at the University of California, at OSU Medical Center. He is board certified in neurology, San Francisco. He finished a second fellowship in bioinformatics clinical neurophysiology, electrodiagnostic medicine, and at the Harvard-Children’s Hospital Informatics Program. Dr. neuromuscular medicine. His research interests include muscular Greenberg is an associate professor of neurology at Harvard dystrophy, spinal muscular atrophy, myasthenia gravis, and other Medical School and on the neurology faculty at Brigham and neuromuscular diseases. Women’s Hospital. He is a faculty member of the Children’s Hospital Informatics Program, where his laboratory is located. His major research interests are understanding mechanisms and developing therapies for autoimmune muscle diseases. He also studies how citation patterns influence scientific belief, research misconduct, and neurological disorders associated with zinc exposure and copper deficiency.

5 Annabel K. Wang, MD Department of Neurology University of California-Irvine Orange, California

Dr. Wang is an associate professor in the Department of Neurology at the University of California-Irvine, the director of its Neuromuscular Diagnostic Laboratory and Neurologist at the VA Long Beach Healthcare System, Long Beach CA. She received her medical degree from McGill University and completed her neurology residency at Tufts New England Medical Center. She has an EMG and neuromuscular fellowship from Beth Israel Hospital and peripheral nerve diseases and neurology research fellowships from Mayo Graduate School of Medicine. Her research interests are peripheral neuropathies, autonomic neuropathies, and myasthenia gravis.

Gregory T. Carter, MD, MS Department of Clinical Neurosciences Providence Medical Group Olympia, Washington

Dr. Carter is medical director of the Muscular Dystrophy Association Regional Neuromuscular Center and the Providence Hospice and Palliative Care program in Olympia, WA. After receiving his medical degree from Loyola University, Carter completed a PMR internship and residency and neuromuscular disease (NMD) fellowship at the University of California, Davis. He completed a pain clinical fellowship and a geriatric medicine mini-fellowship at the University of Washington. His current research focuses on improving rehabilitation management of patients with NMD. He has co-authored more than 150 peer- reviewed publications, 10 book chapters, and dozens of editorials, commentaries, and reviews and serves as senior associate editor for Muscle & Nerve. Dr. Carter is the recipient of the 2012 AANEM Distinguished Researcher Award.

Andrew Tarulli, MD Department of Neurology Beth Israel Deaconess Medical Center Boston, MA

Dr. Tarulli is assistant professor of Neurology at Harvard Medical School and directs the Neuromuscular Medicine fellowship at Beth Israel Deaconess Medical Center in Boston. He also is the associate residency program director and associate medical student clerkship director at Beth Israel Deaconess. Dr. Tarulli received his medical degree from New York University and completed his neurology residency and clinical neurophysiology fellowship at Beth Israel Deaconess. He has authored 23 publications, including Neurology: A Clinician’s Approach, a textbook designed for junior neurology residents. He won an AANEM President’s Research Initiative Award in 2008 and has a strong interest in clinical neuromuscular disease, clinical research, and education.

6 NEUROMUSCULAR UPDATE II

Therapy for Inflammatory Myopathies

Richard J. Barohn, MD Chair of the Department of Neurology University of Kansas Medical Center Kansas City, Kansas

Immunosuppressive therapy is the mainstay of treatment in patients and routes of administration. Prednisone 1 mg/kg/d (60-100 mg) with active disease related to dermatomyositis (DM), polymyositis can be administered for 4 weeks followed by an abrupt or tapered (PM), and necrotizing (NM)1 (Table). Autoimmune NM conversion to an every other day schedule. This taper is slower in is often more resistant to immunosuppressive therapy than DM and patients with severe disease. A daily CS schedule is necessary in PM, particularly if there is an underlying malignancy or a statin well-controlled hypertensive or non-brittle diabetic patients. While trigger. The overwhelming majority (23/25) of statin-associated most patients feel immediately better after taking CS, strength im- necrotizing autoimmune myopathy (SANAM) cases required more provement is delayed by 2-3 months after the onset of treatment. An than one immunosuppressive agent with relapse in 12 cases follow- immediate response may suggest an alternate diagnosis such as poly- ing immunosuppressive therapy tapering.2 However as in DM and rheumatica. For the first 3 months, the typical adult patient PM, immunosuppressants such as prednisone in combination with remains on prednisone 60-100 mg every other day or its equivalent. methotrexate (MTX) or azathioprine (AZA) are the mainstay of If no improvement is noted after 3-6 months, or if weakness reoc- treatment in autoimmune NM. For resistant or severe cases, adding curs during the taper, a second-line immunosuppressive agent such intravenous immunoglobulin (IVIg) may be helpful. Third-line as AZA, MTX or IVIg can be started. These treatments are initiated drugs include mycophenolate mofetil, cyclosporine, tacrolimus, early on with CS therapy in patients with uncontrolled hypertension, rituximab, etanercept, and cyclophosphamide. diabetes, osteoporosis, or obesity and in those with baseline severe weakness. For good responders, a taper by 20 mg/month until 40 mg The few published randomized controlled trials of immunosuppres- every other day, then by 10 mg/month, will reduce the prednisone sion in DM or PM compared placebo to AZA,3 plasma exchange,4 or dose to 20 mg every other day after 6-8 months from the initiation IVIg.5 In addition, randomized controlled trials compared MTX with of therapy. After that, the taper is by 5 mg, and the interval is every AZA,6 cyclosporine with MTX,7 and IV MTX with oral MTX plus 3 months to reach the minimal effective dose. In severe cases, the AZA.3,6-8 The only positive-controlled trials are a small cross-over author prefers starting with a 5-day IV pulse methyprednisolone study of IVIg in DM5 and a randomized, double-blind, placebo- therapy followed by high-dose oral prednisone in combination with controlled trial of etanercept (50 mg subcutaneously weekly) for 52 a second-line drug. weeks in 16 DM subjects.9 Recently, a randomized multicenter double-blind clinical trial com- CORTICOSTEROIDS pared oral dexamethasone pulse therapy to daily prednisolone in 62 patients with subacute onset myositis.10 The pulsed regimen While no controlled trial has been performed using corticosteroids consisted of dexamethasone given as six cycles of 40 mg/day for 4 (CS), there is general agreement that these are effective in DM, PM, consecutive days at 28-day intervals. While pulsed high-dose oral and NM. Corticosteroids can be used in a wide range of regimens dexamethasone was not found to be superior to daily prednisolone as

7 THERAPY FOR INFLAMMATORY MYOPATHIES

Table. Immunosuppressive therapy for inflammatory myopathies

Therapy Route Dose Side effects Monitor

Flu-like illness, hepatoxicity, pancreatitis, leucopenia, Azathioprine By mouth 2-3 mg/kg/day; single a.m.dose Monthly blood count, liver enzymes macrocytosis, neoplasia, infection, teratogenicity

Bone marrow suppression, hepatoxicity, neoplasia, Chlorambucil By mouth 4-6 mg/day, single a.m. dose Monthly blood count, liver enzymes infertility, teratogenicity, infection

Bone marrow suppression, infertility, hemorrhagic cystitis, By mouth 1.5-2 mg/kg/day; single a.m. dose Monthly blood count, urinalysis alopecia, infections, neoplasia, teratogenicity Cyclophosphamide Same as by mouth (although more severe), and IV 1 gm/m2 Daily to weekly blood count, urinalysis nausea/vomiting, alopecia

Nephrotoxicity, hypertension, infection, hepatoxicity, Blood pressure, monthly cyclosporine level, Cyclosporine By mouth 4-6 mg/kg/day, split into two daily doses hirsutism, tremor, gum hyperplasia, teratogenicity creatinine/BUN, liver enzymes

Hypotension, arrhythmia, diaphoresis, flushing, Intravenous 2 gm/kg over 2-5 days; then every 4-8 weeks as IV nephrotoxicity, headache, nausea, aseptic meningitis, Heart rate, blood pressure, creatinine/BUN immunoglobulin needed anaphylaxis, stroke Hepatoxicity, pulmonary fibrosis, infection, neoplasia, 7.5-20 mg weekly, single or divided doses; 1 day Monthly liver enzymes, blood count; consider liver biopsy By mouth infertility, leucopenia, alopecia, gastric irritation, stomatitis, a week dosing at 2 gm accumulative dose Methotrexate teratogenicity IV/IM 20-50 mg weekly; 1 day a week dosing Same as by mouth Same as by mouth

1 gm in 100 ml/normal saline over 1-2 hours, Arrhythmia, flushing, dysgeusia, anxiety, insomnia, fluid Methylprednisone IV Heart rate, blood pressure, serum glucose/potassium daily or every other day for 2-6 doses and weight gain, hyperglycemia, hypokalemia, infection

Myelosuppression, GI (diarrhea, nausea, abdominal pain), Mycophenylate By mouth 1-1.5 gm twice a day peripheral edema, fever, infection, opportunistic infection, Monthly blood count mofetil malignancy, teratogenicicity

Hypertension, fluid and weight gain, hyperglycemia, 60-100 mg/day for 2-4 weeks, then 100 mg every Weight, blood pressure, serum glucose/potassium, cataract Prednisone By mouth hypokalemia, cataracts, gastric irritation, osteoporosis, other day; single a.m. dose formation infection, aseptic femoral necrosis

Two doses of 750 mg/m2 administered 2 weeks Mild infusion-related adverse events (headache, nausea, CD19 counts (< 5%), IgG level (keep above 30% of the Rituximab IV apart chills, hypotension), anaphylaxis, infection lower normal limit)

Nephrotoxocity, GI (diarrhea, abdominal pain), Blood pressure, creatinine/BUN, and electrolytes, monthly Tacrolimus By mouth 0.1-0.2 mg/kg/day split into two daily doses hypertension, electrolyte imbalance, tremor, infection, trough level (aim 5-15 ng/ml) hepatotoxicity, teratogenicity

BUN = blood urea nitrogen, GI = gastrointestinal, IgG = immunoglobulin G, IM = intramuscular, IV = intravenous first-line treatment of idiopathic inflammatory myopathies, it caused Patients and their families are asked to be alert about personality substantially fewer side effects. Ten patients (33%) treated with pred- changes and psychiatric side effects. Patients are also directed to nisolone and one patient (4%) treated with pulsed dexamethasone reduce the salt and carbohydrate in their diet and visit regularly with developed diabetes mellitus. Mood changes occurred in 20 patients the primary care physician for blood pressure, serum glucose and po- (67%) and eight (29%), respectively. Treatment adjustments for co- tassium, and measurements. The author advocates the pneumococcal morbid conditions (hypertension, diabetes mellitus) were needed in vaccine and yearly flu shots. Given the immunosuppressed state, evi- one patient in the dexamethasone group, compared with 15 patients dence indicates that seroprotection of the influenza A/H1N1 vaccine in the prednisolone group. However, there was a large number of is significantly reduced DM patients compared to control subjects.12 early disconstinuations in both groups (21 and 17, respectively) for In a retrospective study of 279 PM/DM cases over 15 years, 37% a variety of reasons. were admitted for a severe pyogenic infection (n = 71) mostly due to aspiration pneumonia or for an opportunistic infection (n = 33).13 Because the risks of longterm CS therapy are numerous, discuss- There are currently no consensus criteria to identify patients who are ing those with the patient as well as establishing a monitoring plan at high risk for Pneumocystis carinii pneumonia (PCP) infection and in collaboration with the primary care physician is integral to the would therefore benefit from prophylaxis. Patients with total lym- management plan. Before CS initiation, a purified protein derivative phocyte counts < 800/μL and/or CD4 lymphocyte counts < 200-400/ skin test can identify the need for isoniazid in previously exposed μL are likely to benefit from PCP prophylaxis prior to initiation of cases. As CS is started, a baseline bone dual-energy X-ray absorp- and in the course of immunosuppressive therapy.14 tiometry scan is obtained and the patient is requested to seek an opthtalmologic examination, with yearly followup for both. Patients METHOTREXATE are maintained on oral calcium 500-600 mg two to three times daily with vitamin D 400 IU daily. In juvenile DM (JDM), the incidence Methotrexate, an antifolate that inhibits lymphocyte proliferation, of vertebral fracture 12 months following steroid initiation was 6% is an effective more rapidly acting second-line steroid-sparing im- and these were mostly asymptomatic.11 The risk of fracture increased munosuppressant. Oral MTX can be started at 7.5 mg/week, and in with higher steroid dose, and in the first 6 months with greater in- 2 weeks increased to 15 mg/week in two divided doses. The dose is creases in body mass index or greater declines in spine Z scores. then increased by 2.5 mg/week at 3 months, depending on response 8 NEUROMUSCULAR UPDATE II to reach the maximum weekly dose of 25 mg. Daily folic acid The dose is adjusted to treatment response and to maintain the white 0.8-1.0 mg orally is also administered to prevent stomatitis. Some cell count above 3,500 and the absolute lymphocyte count below advocate skipping folic acid on the day of MTX administration. 1,000. If the white cell count falls below 2,500 or the absolute neutrophil count is below 1,000, AZA administration must be inter- Besides stomatitis, potential adverse events include alopecia, pneu- rupted. Patients taking allopurinol, an inhibitor of the main detoxifi- monitis, teratogenicity, induction of malignancy, susceptibility to cation pathway, require AZA dose reduction to 25-33% of the above. infections, and renal insufficiency. For bone marrow suppression Angiotensin converting enzyme inhibitors must be avoided due to and liver toxicity, complete blood count, differential count, and the serious risk of severe leucopenia. liver function tests are monitored weekly in the first 4 weeks, then monthly for 6 months, and every 3 months thereafter while on a Intravenous Immunoglobulin stable dose. Methotrexate-induced pneumonitis can be difficult to distinguish from myositis-associated interstitial lung disease (ILD). Intravenous immunoglobulin, a pooled gammaglobulin product It is not used in patients with known ILD or in those with the Jo-1 from several thousand blood donors, has a complex immunomodu- antibodies. latory mechanism of action. Potential mechanisms include modula- tion and inhibition of pathogenic autoantibodies; suppression of Therapeutic effects of oral MTX are often noticeable after 4-8 proinflammatory cytokines; blockade of Fc receptor, macrophage weeks. If no improvement is observed by that time, the dose is esca- colony stimulating factor, and monocyte chemotactant protein-1 in- lated. In nonresponders and in more severe cases, the author recom- crease; alteration of T cell function; suppression of circulating CD54 mends MTX IV or intramuscular treatment at a dose of 0.4-0.8 mg/ lymphocytes; and inhibition of cell transmigration into the muscle. kg weekly injections, increasing it by 5 mg every week to reach up A randomized-controlled trial with optional cross-over showed that to 60 mg weekly. Leucovorin rescue on the day after parenteral MTX IVIg 2 gm/kg administered monthly for 3 months was very effective administration is needed for doses as high as 50 mg. (9/12) in treatment-resistant DM.5 Though prospective controlled trials are lacking, IVIg is also believed to be effective in PM17,18 Azathioprine and NM. The American Academy of Neurology 2012 guidelines recommend IVIg as possibly effective and to be considered for treat- Azathioprine, an anti-metabolite that blocks T-lymphocyte prolifera- ing nonresponsive DM cases.19 There was insufficient evidence to tion, is a very effective second-line steroid-sparing immunosuppres- support or refute the use of IVIg in PM or inclusion body myositis. sant with delayed onset of response. It is administered in divided In a recent retrospective inception cohort, all 78 JDM cases were doses of 2-3 mg/kg/day, ranging 100-250 mg/day. Start with 50 mg/ treated steroids but 30 were treated additionally with IVIg.20 The day for a week before gradually increasing the dose over 1-2 weeks IVIg group maintained similar or lower disease activity than control to 100-150 mg/day. In 3-6 months, the author may increase it to the subjects. To achieve a more rapid improvement in severely affected maximum range of 200-250 mg/day. Onset of response is delayed to patients IVIg in instituted in addition to IV solumedrol as initial at least 4-8 months and peaks at 1-2 years. It is therefore not surpris- therapy. Occasionally, the author will administer IVIg as mainte- ing that the 3-month placebo controlled trial of AZA did not show nance therapy in otherwise refractory patients, or more commonly any efficacy.3 However, hand grip strength improvement after 1 year to reduce long-term CS dose. Dosing is 2 g/kg total initially, given was no different when comparing the AZA to MTX recipients.6 Prior divided over 2-5 days, and then infusions are repeated every 2-4 to AZA initiation, it has recently been suggested to test for thiopurine weeks, with a total monthly dosage of 0.4-2.0 g/kg. methyltransferase activity. Its deficiency predicts an increased risk of hematologic toxicity. In homozygous cases AZA is contraindi- Patients are closely monitored with the first infusion, starting at a cated in while in heterozygous patients lower doses may be care- very slow rate of 25-50 cc/hr for 30 min, which is increased pro- fully tried.15,16 A metanalysis of 54 observational studies and one gressively by 50 cc/hr every 15-20 min up to 150-200 cc/hr. Mild randomized controlled trial did not demonstrate sufficient evidence reactions (headache, nausea, chills, myalgia, chest discomfort, and to address the effectiveness of thiopurine methyltransferase activity back pain) occur in 10% and are improved with slowing the infusion pretesting. In clinical practice, the author monitor spatients’ blood rate and are preventable with premedication with acetaminophen, cell counts weekly at the initiation of AZA, then monthly.15 A flu- benadryl, and if need be IV methylprednisolone. Moderate rare re- like reversible acute hypersensitivity reaction affects 12% of users actions include chemical meningitis and delayed red, macular skin in the first 2 weeks of therapy. It is associated with a rash, elevation reaction of the palms, soles, and trunk with desquamation. Acute in liver enzymes, and pancreatitis. Some may tolerate a re-challenge. renal failure is uncommon and related to patient dehydration and Delayed adverse events include myelosuppression, hepatotoxicity, the sucrose or maltose diluent. Other severe and rare reactions are suseptibility to infection, malignancy, teratogenicity, rash, alopecia, anaphylaxis, stroke, myocardial infarction, or pulmonary emboli due fever, and arthralgias. to hyperviscocity syndrome. The latter is more likely to occur in old age, immobility, diabetes, thrombocytemia, hypercholesterolemia, As long as the patient remains on stable AZA doses, CBC and liver hypergammaglobunemia, and cryoglobunemia. Avoid using IVIg in enzymes should be monitored every week for 4 weeks, then monthly patients with several of these risk factors and place IVIg recipients for 6 months, and then, for the risk for delayed toxicity, every 3 on low-dose aspirin prophylactically. The extremely rare patients months. When liver enzymes are markedly elevated (above two with total IgA deficiency should not be receiving IVIg. In an uncon- times the normal limit), AZA should be stopped for several months trolled preliminary report, seven Caucasian women (four with DM until enzymes normalize before the patient may be re-challenged, at and three with PM) with median disease duration of 72 months were times successfully. It is important to obtain levels of liver-specific treated with subcutaneous IG.20 Over a median followup period of gamma-glutamyl transferase since transaminases may be released 14 months, all patients showed a favorable clinical response leading from necrotic muscle fibers. 9 THERAPY FOR INFLAMMATORY MYOPATHIES to reduction of the daily maintenance prednisone dose by a mean IDIOPATHIC INFLAMMATORY MYOSITIS of 23 mg and three patients were able to discontinue prednisone ASSOCIATED WITH INTERSTITIAL LUNG altogether.21 These encouraging findings have not been not been DISEASE validated in a controlled study. Corticosteroids are the first-line drug for idiopathic inflammatory Refractory Patients myositis (IIM) associated with ILD, but most patients require an adjuvant immunomodulating drug.32 In cases of ILD refractory to In refractory patients, mycophenylate mofetil22 is started as a third- steroids, mycophenolate mofetil,33 cyclosporine, and tacrolimus have line agent and, in severe cases, rituximab23,24 or cyclophosphamide as been shown to be effective second-line agents.25 Early intervention well. Other third-line drugs include etanercept,9 cyclosporine, tacro- with prednisolone and cyclosporin A (CsA) combination therapy limus,25 and chlorambucil26 (Table). A large multicenter clinical trial and tight control of the daily CsA dose (by monitoring the blood to clarify the role of CD20 depletion using rituximab in refractory level 2 hours post-dosing) improved pulmonary function test and PM and DM adults and children was conducted. The results were chest imaging findings in DM cases with acute-to-subacute ILD.34 presented at the 2010 American College of Rheumatology Annual Rituximab and cyclophosphamide are third-line options to arrest meeting and the article is currently in press.27 Two-hundred patients progression in cases of refractory ILD. A third of treated cases with DM, JDM, or PM were randomized to receive rituximab early experience resolution of pulmonary involvement, whereas 16% de- (Group A) or late (8 weeks later, Group B) in the course of this teriorated.35 Factors predictive of poor ILD prognosis include older 44-week trial. The primary endpoint measure compared the time to age, symptomatic ILD, lower values of vital capacity and diffusing achieve the definition of improvement (DOI) on two consecutive capacity for carbon monoxide, a pattern of interstitial pneumonia visits between both groups. The secondary endpoint measure was the on high resolution computed tomography scan and lung biopsy, and difference in time to achieve 20% improvement on manual muscle steroid-refractory ILD. There is increased mortality rate in patients testing on two consecutive visits between both groups. Another with deteriorating ILD as compared to those without ILD deteriora- secondary endpoint measure was to compare between groups the tion (47.1% versus 3.3%). proportion of patients achieving the DOI at week 8. There were no significant differences between Group A and B in the primary (20.2 PROMISING THERAPIES and 20.0 weeks, respectively) and in the secondary endpoints since the proportion of early and late rituximab patients achieving DOI at Tocilizumab, a humanized anti-IL-6 receptor antibody, is Food and week 8 was 21% and 15%, respectively (p = 0.32). It is likely that the Drug Administration (FDA)-approved for the treatment of moderate- study-delayed treatment design hampered the detection of a signifi- to-severe rheumatoid arthritis.36 Ogata and Tanaka recently reported cant benefit of rituximab as 83% of refectory cases met the DOI fol- the first two cases of refractory PM who responded to tocilizumab lowing rituximab treatment. In a small uncontrolled study, rituximab with reduction in CK as well as in one of the cases resolution of improved six of eight refractory signal recognition particle-positive myalgia and stabilization of disease activity and in the other disap- patients on manual muscle strength and/or resulted in creatine kinase pearance of the high-intensity zones in the thigh muscles on magnetic (CK) decline as early as 2 months after treatment.28 Quantitative resonance imaging.36 Abatacept, a fusion protein between Ig and the levels of serum anti-SRP antibodies also decreased after rituximab extracellular domain of cytotoxic T-lymphocyte antigen 4, exerts its treatment. Furthermore, anti-SRP-positive myositis appears to be anti-inflammatory effect by down-regulating T-cell activation and one of the few autoimmune diseases in which specific autoantibody is FDA-approved for moderate-to-severe rheumatoid arthritis. A levels are correlated with surrogate disease activity markers.29 refractory JDM case responded to abatacept IV and topical sodium thiosulfate (which promotes vasodilation and vascularization) with In a controlled trial of etanercept, five of 11 subjects in the etaner- significant reductions in muscle and skin inflammation, decreased cept arm were successfully weaned off prednisone whereas none of corticosteroid dependence, and an arrest of calcinosis progression.36 the five placebo-recipients could be weaned off.9 The median of the A bedridden PM patient had a very good clinical response to abata- average prednisone dosage after week 24 was lower in the etanercept cept.37,38 Therefore, abatacept may hold promise as steroid-sparing group (1.2 mg/day) than in the placebo group (29.2 mg/day). Five agent for the treatment of refractory DM. etanercept-treated subjects and one placebo-treated case developed worsening DM rash. Although a case of refractory DM-ILD was There are anecdotal reports of immune ablation similar to that successfully treated with adalimumab,30 another case of rheumatoid achieved following myeloablative autologous hematopoietic stem arthritis (RA) developed DM 4.5 years after treatment with this cell transplantation through the intensive administration of alem- anti-tumor necrosis factor (TNF)-α drug.31 The TNF antagonists are tuzumab, an anti-CD52 antibody. A patient with refractory PM re- widely used for the treatment of rheumatoid arthritis (RA), as well sponded rapidly to a single course of treatment with alemtuzumab.39 as for inflammatory bowel disease, psoriasis, psoriatic arthritis, and Another case of refractory juvenile PM treated with alemtuzumab ankylosing spondylitis. Lupus-like symptoms have been reported in had stable clinical improvement for more than 6 years.40 However, a < 0.2% of patients within 4-9 months of treatment for these diseases, refractory adult PM case developed Epstein-Barr virus-driven lym- but development of PM/DM is limited to patients with underly- phoproliferative disorder 9 weeks after alemtuzumab therapy.41 In ing RA. It is likely in the latter case that DM is a manifestation of a preliminary report, 10 patients with refractory or severe DM/PM overlap syndrome rather it being induced by adalimumab. Until an underwent allogeneic mesenchymal stem cell transplantation and all association of PM/DM with TNF antagonists is demonstrated in initially improved in their serum CK, patient global assessment by patients who do not have RA, it is premature to ascertain a causal visual analogue scale, and muscle strength by manual muscle test.42 relationship. However, three had recurrence of disease activity 6-8 months after the transplant of whom two underwent a second transplant with good 10 NEUROMUSCULAR UPDATE II clinical response in one case. The efficacy of allogeneic mesenchy- CADM-140 antibodies). Complete remission of PM/DM was less mal stem cell transplantation in DM or PM has yet to be confirmed frequent (13.6% versus 41.1%) and the mortality rate (47.8% versus in controlled trials. 7.3%) was higher in elderly patients than in younger patients.53 In a recent series, the coexistence of Ro52 and Jo1 antibodies was associ- Recently an old therapy, intramuscular ACTH, has been reported ated with more severe myositis/joint impairment, symptomatic ILD, to be effective for inflammatory myopathy.42a This drug is surpris- increased risk of cancer, and higher mortality.56 Anti-SRP antibody ingly FDA-approved for the treatment of dermatomyositis and is associated with acute onset of refractory necrotizing myositis and polymyositis. antibody titers correlate with CK levels and disease activity.57 Anti- 155/140 antibody is associated with malignancy, whereas the pres- PHYSICAL THERAPY ence of anti–CADM-140 antibody is associated with amyopathic DM and rapidly progressive ILD. In addition to pharmacotherapy, it is generally agreed upon that physical therapy, orthotic devices, occupational therapy, and ex- ercise are beneficial in DM, PM, and NM as early as 2-3 weeks from the acute phase.43 While other studies have reported the safety and benefits of resistive exercise in active patients 1-3 months into their treatment,44,45 most of the studies have been in chronic PM or DM.45 All studies demonstrated the efficacy and safety of exercise as measured by the Functional Index, SF-36, muscle histology, muscle magnetic resonance imaging scanning, or creatine phosphokinase levels. Besides improved muscle strength and increased maximal oxygen uptake, resistance exercise training of eight myositis patients resulted in marked reductions in gene expression, reflecting reduc- tions in proinflammatory and profibrotic gene networks, together with a reduction in tissue fibrosis.46 In severe cases, the author will start with passive range of motion exercises initially and will usually wait for the first month to 3 months for strength and CK to start re- sponding to pharmacotherapy before subjecting severely weakened muscles to a rigorous strengthening exercise program. In patients with mild-to-moderate weakness, the author will initiate a strength- ening program after 2-4 weeks of steroid initiation. Since pain from arthralgia and possibly arthritis is relieved by joint flexion, early mo- bilization is important to prevent flexion of the large and small joints, especially in JDM. There may also be a role for creatine monohydate supplementation as it improves functional performance without significant adverse effects.47

PROGNOSIS

The prognosis of DM, PM, and NM is in general favorable with some exceptions. An associated malignancy portends a poor prognosis for recovery and increases mortality. SANAM is resistant to treatment. Concomitant ILD or Jo-1 or signal recognition particle autoantibod- ies predict a poorer prognosis. Overall, drug-free remissions are rare except in JDM. Recent series underline that only 20-40% of treated patients will achieve PM/DM remission, whereas 60-80% will expe- rience a polycyclic or chronic continuous course of the disease.48,49 On medium- and longterm followup, up to 80% of treated PM/DM patients are still disabled based on Health Assessment Questionnaire scores.50 The overall mortality ratio in PM/DM patients also remains two- to threefold higher compared with the general population, with cancer, lung, and cardiac complications and infections being the most common causes of death.51,52 Poor prognostic factors in PM/DM patients include older age,53 male gender, non-Caucasian ethnicity, longer symptoms duration, ILD,54 cardiac involvement, dysphagia,55 cancer,52 and serum myositis-specific antibodies (in- cluding coexistence of anti-Ro52 and anti-Jo1 antibodies, presence of anti-signal recognition particle antibody, anti-155/140, and anti-

11 THERAPY FOR INFLAMMATORY MYOPATHIES REFERENCES 20. Lam CG, Manlhiot C, Pullenayegum EM, Feldman BM. Efficacy of intravenous Ig therapy in juvenile dermatomyositis. Ann Rheum Dis 2011;70(12):2089-2094. 1. Amato AA, Barohn RJ. Evaluation and treatment of inflammatory 21. Danieli MG, Pettinari L, Moretti R, Logullo F, Gabrielli A. myopathies. J Neurol Neurosurg Psych 2009;80:1060-1068. Subcutaneous immunoglobulin in polymyositis and dermatomyositis: 2. Grable-Esposito P, Katzberg HD, Greenberg SA, Srinivasan J, Katz J, a novel application. Autoimmun Rev 2011;10(3):144-149. Amato AA. Immune-mediated necrotizing myopathy associated with 22. Rowin J, Amato AA, Deisher N, Cursi, J, Meriggioli MN. statins. Muscle Nerve 2010;41(2):185-190. Mycophenolate mofetil in dermatomyositis: is it safe? Neurology 3. Bunch TW, Worthington JW, Combs JJ, Ilstrup MS, Engel AG. 2006;66(8), 1245-1247. Azathioprine with prednisone for polymyositis. Ann Intern Med 23. Levine TD. Rituximab in the treatment of dermatomyositis: an open- 1980;92(3):365-369. label pilot study. —Arthritis Rheum 2005;52(2):601-607. 4. Miller FW, Leitman SF, Cronin M, et al. Controlled trial of plasma 24. Noss EH, Hausner-Sypek DL, Weinblatt ME. Rituximab as therapy exchange and leukapheresis in polymyositis and dermatomyositis. N for refractory polymyositis and dermatomyositis. J Rheumatol Engl J Med 1992;326:1380-1384. 2006;33(5):1021-1026. 5. Dalakas MC, Illa I, Dambrosia JM, et al. A controlled trial of high-dose 25. Oddis CV, Sciurba FC, Elmagd KA, Starzl TE. Tacrolimus in intravenous immune globulin infusions as treatment for dermatomyosi- refractory polymyositis with interstitial lung disease. Lancet tis. N Engl J Med 1993;329(27):1993-2000. 1999;22;353(9166):1762-1763. 6. Miller J, Walsh Y, Saminaden S, Lecky BRF, Winer JB. Randomized 26. Cagnoli M, Marchesoni A, Tosi S. Combined steroid, methotrexate and double blind controlled trial of methotrexate and steroids compared chlorambucil therapy for steroid-resistant dermatomyositis. Clin Exp with azathioprine and steroids in the treatment of idiopathic inflamma- Rheumatol 1991;9(6):658-659. tory myopathy. J Neurol Sci 2002;199(Suppl 1):S53. 27. Oddis CV, Reed AM, Aggarwal R, et al. The RIM study group: 7. Vencovsky J, Jarosova K, Machacek S, et al. Cyclosporine A versus Rituximab in the treatment of refractory adult and juvenile dermato- methotrexate in the treatment of polymyositis and dermatomyositis. myositis (DM) and adult polymyositis (PM)—The RIM study. Arthritis Scand J Rheum 2000;29(2):95-102. Rheum 2010:62:3844. 8. Villalba L, Hicks JE, Adams EM, et al. Treatment of refractory myosi- 28. Valiyil R, Casciola-Rosen L, Hong G, Mammen A, Christopher-Stine tis: a randomized crossover study of two new cytotoxic regimens. Arth L. Rituximab therapy for myopathy associated with anti-signal recog- Rheum 1998;41(3):392-399. nition particle antibodies: a case series. Arthritis Care Res (Hoboken) 9. Muscle Study Group. A randomized, pilot trial of etanercept in derma- 2010;62(9):1328-1334 tomyositis. Ann Neurol 2011;70(3):427-436. 29. Benveniste O, Drouot L, Jouen F, et al. Correlation of anti-signal recog- 10. van de Vlekkert J, Hoogendijk JE, de Haan RJ, et al. Oral dexa- nition particle autoantibody levels with creatine kinase activity in patients methasone pulse therapy versus daily prednisolone in sub-acute with necrotizing myopathy. Arthritis Rheum 2011;63(7):1961-1971. onset myositis, a randomised clinical trial. Neuromuscul Disord 30. Park JK, Yoo HG, Ahn DS, Jeon HS, Yoo WH. Successful treatment 2010;20(6):382-389. for conventional treatment-resistant dermatomyositis-associated in- 11. Rodd C, Lang B, Ramsay T, et al. Incident vertebral fractures among terstitial lung disease with adalimumab. Rheumatol Int 2011 Nov 17. children with rheumatic disorders 12 months after glucocorticoid ini- Epub ahead of print. tiation: a national observational study. Arthritis Care Res (Hoboken) 31. Brunasso AM, Scocco GL, Massone C. Dermatomyositis during adalim- 2012;64(1):122-131. umab therapy for rheumatoid arthritis. J Rheumatol 2010;37(7):1549-1550. 12. Saad CG, Borba EF, Aikawa NE, et al. Immunogenicity and safety of 32. Mimori T, Nakashima R, Hosono Y. Interstitial Lung Disease the 2009 non-adjuvanted influenza A/H1N1 vaccine in a large cohort in Myositis: Clinical Subsets, Biomarkers, and Treatment. Curr of autoimmune rheumatic diseases. Ann Rheum Dis 2011;70(6):1068- Rheumatol Rep 2012 Feb 25. Epub ahead of print. 1073. 33. Morganroth PA, Kreider ME, Werth VP. Mycophenolate mofetil 13. Marie I, Ménard JF, Hachulla E, et al. Infectious complications in poly- for interstitial lung disease in dermatomyositis. Arthritis Care Res myositis and dermatomyositis: a series of 279 patients. Semin Arthritis (Hoboken) 2010;62(10):1496-1501. Rheum 2011;41(1):48-60. 34. Kotani T, Takeuchi T, Makino S, et al. Combination with corticoste- 14. Efthimiou P, Pokharna H, Kukar M, Hennessey K. PCP chemoprophy- roids and cyclosporin-A improves pulmonary function test results and laxis is essential for lymphopenic dermatomyositis patients treated with chest HRCT findings in dermatomyositis patients with acute/subacute immunomodulators. Muscle Nerve 2011;43(6):918-919. interstitial pneumonia. Clin Rheumatol 2011;30(8):1021-1028. 15. Evans WE, Hon YY, Bomgaars L, et al. Preponderance of thio- 35. Marie I, Hatron PY, Dominique S, Cherin P, Mouthon L, Menard JF. purine S-methyltransferase deficiency and heterozygosity among Short-term and long-term outcomes of interstitial lung disease in poly- patients intolerant to mercaptopurine or azathioprine. J Clin Oncol myositis and dermatomyositis: a series of 107 patients. Arthritis Rheum 2001;19(8):2293-2301. 2011;63(11):3439-3447. 16. Booth RA, Ansari MT, Loit E, et al. Assessment of thiopurine 36. Ogata A, Tanaka T. Tocilizumab for the treatment of rheumatoid arthri- S-methyltransferase activity in patients prescribed thiopurines: a sys- tis and other systemic autoimmune diseases: current perspectives and tematic review. Ann Intern Med 2011;154(12):814-823. future directions. Int J Rheumatol 2012;2012:946048. 17. Cherin P, Piette JC, Wechsler B, et al. Intravenous gamma globulin as 37. Arabshahi B, Silverman RA, Jones OY, Rider LG. Abatacept and first line therapy in polymyositis and dermatomyositis: an open study sodium thiosulfate for treatment of recalcitrant juvenile dermatomyositis in 11 adult patients. J Rheum 1994;21:1092-1097. complicated by ulceration and calcinosis. J Pediatr 2012;160(3):520-522. 18. Danieli MG, Malcangi G, Palmieri C, et al. Cyclosporin A and intrave- 38. Musuruana JL, Cavallasca JA. Abatacept for treatment of refractory nous immunoglobulin treatment in polymyositis/dermatomyositis. Ann polymyositis. Joint Bone Spine 2011;78(4):431-432. Rheum Dis 2002;61:37-41. 39. Thompson B, Corris P, Miller JA, Cooper RG, Halsey JP, Isaacs JD. 19. Patwa HS, Chaudhry V, Katzberg H, Rae-Grant AD, So YT. Evidence- Alemtuzumab (Campath-1H) for treatment of refractory polymyositis. based guideline: Intravenous immunoglobulin in the treatment of J Rheumatol 2008;Oct;35(10):2080-2082. neuromuscular disorders: report of the Therapeutics and Technology Assessment Subcommittee of the American Academy of Neurology. Neurology 2012;27;78(13):1009-1015.

12 NEUROMUSCULAR UPDATE II 40. Reiff A, Shaham B, Weinberg KI, Crooks GM, Parkman R. Anti-CD52 48. Marie I, Hachulla E, Hatron PY, et al. Polymyositis and dermato- antibody-mediated immune ablation with autologous immune recovery myositis: short term and longterm outcome, and predictive factors. J for the treatment of refractory juvenile polymyositis. J Clin Immunol Rheumatol 2001;28:2230-2237. 2011;31(4):615-622. 49. Bronner IM, van der Meulen MF, de Visser M, et al. Long-term 41. Cooles FA, Jackson GH, Menon G, Isaacs JD. Epstein-Barr virus-driven outcome in polymyositis and dermatomyositis. Ann Rheum Dis lymphoproliferative disorder post-CAMPATH-1H (alemtuzumab) in 2006;65:1456-1461. refractory polymyositis. Rheumatology (Oxford) 2011;50(4):810-812. 50. Ponyi A, Borgulya G, Constantin T, et al. Functional outcome and 42. Wang D, Zhang H, Cao M, et al. Efficacy of allogeneic mesenchymal quality of life in adult patients with idiopathic inflammatory myositis. stem cell transplantation in patients with drug-resistant polymyositis Rheumatology (Oxford) 2005;44:83-88. and dermatomyositis. Ann Rheum Dis 2011;70(7):1285-1288. 51. Limaye V, Hakendorf P, Woodman RJ, Blumbergs P, Roberts-Thomson 42a. Levine T. Treating refractory dermatomyositis or polymyositis with P. Mortality and its predominant causes in a large cohort of pa- adrenocorticotropic hormone gel: a retrospective case series. Drug Des tients with biopsy-determined inflammatory myositis. Intern Med J Devel Ther 2012;6:133-139. 2012;42(2):191-198. 43. Varjú C, Pethö E, Kutas R, Czirják L. The effect of physical exercise 52. Airio A, Kautiainen H, Hakala M. Prognosis and mortality of polymyo- following acute disease exacerbation in patients with dermato/poly- sitis and dermatomyositis patients. Clin Rheumatol 2006;25:234-239. myositis. Clin Rehabil 2003;17(1):83-87. 53. Marie I, Hatron PY, Levesque H, et al. Influence of age on charac- 44. Alexanderson H, Stenström CH, Jenner G, Lundberg I. The safety of teristics of polymyositis and dermatomyositis in adults. Medicine a resistive home exercise program in patients with recent onset active (Baltimore) 1999;78:139-147. polymyositis or dermatomyositis. Scand J Rheumatol 2000;29(5):295-301. 54. Yamasaki Y, Yamada H, Ohkubo M, et al. Longterm survival and asso- 45. Alexanderson H. Exercise effects in patients with adult idiopathic ciated risk factors in patients with adult-onset idiopathic inflammatory inflammatory myopathies. Curr Opin Rheumatol 2009;21(2):158-163. myopathies and amyopathic dermatomyositis: experience in a single 46. Nader GA, Dastmalchi M, Alexanderson H, et al. A longitudinal, in- institute in Japan. J Rheumatol 2011;38:1636-1643. tegrated, clinical, histological and mRNA profiling study of resistance 55. Danko K, Ponyi A, Constantin T, et al. Long-term survival of pa- exercise in myositis. Mol Med 2010;16(11-12):455-464. tients with idiopathic inflammatory myopathies according to clinical 47. Chung YL, Alexanderson H, Pipitone N, et al. Creatine supplements features: a longitudinal study of 162 cases. Medicine (Baltimore) in patients with idiopathic inflammatory myopathies who are clini- 2004;83:35-42. cally weak after conventional pharmacologic treatment: Six-month, 56. Marie I, Hatron PY, Dominique S, et al. Short-Term and Long-Term double-blind, randomized, placebo-controlled trial. Arthritis Rheum Outcome of Anti-Jo1-Positive Patients with Anti-Ro52 Antibody. 2007;57(4):694-702. Semin Arthritis Rheum 2011 Nov 9. Epub ahead of print. 57. Benveniste O, Drouot L, Jouen F, et al. Correlation of anti-signal rec- ognition particle autoantibody levels with creatine kinase activity in pa- tients with necrotizing myopathy. Arthritis Rheum 2011;63:1961-1971.

13 14 NEUROMUSCULAR UPDATE II

Inflammatory Myopathies

Steven A. Greenberg, MD Associate Professor of Neurology Department of Neurology, Division of Neuromuscular Disease Brigham and Women’s Hospital and Children’s Hospital Informatics Program Harvard Medical School Boston, Massachusetts

CASE PRESENTATION

Case One Case Two

A 35-year-old woman developed difficulty arising from a chair, A 67-year-old man developed slowly progressive difficulty climbing stairs, washing her hair, and removing items from walking with a tendency for his knees to buckle, difficulty climb- shelves in her closet. Symptoms progressed in intensity for 3 ing stairs, and difficulty holding eating utensils and turning keys. months and she then developed a fine pruritic rash along her Serum CK was 845 IU/L. He underwent muscle biopsy, was upper anterior chest and dry, cracking hands with discoloration diagnosed with polymyositis (PM), and treated with prednisone on the back of her fingers. Physical examination showed an ery- 60 mg orally daily. Three months later followup CK was 252 thematous rash around the eyes and on the anterior chest, purplish IU/L but there was no improvement in strength. He continued to discoloration of papules on the back of the hands, particularly weaken, while remaining on high-dose prednisone, which was over the joints, and moderate weakness of neck flexion, shoulder eventually tapered 1 year later. He underwent another consulta- abduction, arm flexion, hip flexion, and knee flexion. She was tion, which showed generalized weakness with a marked prefer- unable to arise from a chair without using her hands to push ence for wrist and finger flexors (Figure 2), and quadriceps, with off. Laboratory studies showed a normal serum creatine kinase relatively spared arm abductors. He underwent repeat muscle (CK) (63 IU/ml). Complete blood count, electrolytes, antinuclear biopsy (Figure 3). antibodies (ANA), and aspartate aminotransferase/alanine amino- transferase were normal. The muscle biopsy is shown in Figure 1.

Figure 2. Preferential involvement of finger flexors over finger extensors in Case Two. Used with permission from the author. Figure 1. Hematoxylin and eosin stained sections from muscle biopsy Case One. Used with permission from the author. 15 INFLAMMATORY MYOPATHIES a connective tissue disorder such as mixed connective tissue disease), granulomatous myositis, and eosinophilic myositis. The mechanisms initiating and maintaining these diseases are not well understood.6

Dermatomyositis

Dermatomyositis affects children and adults. Adult DM generally presents as subacute progressive painless proximal weakness, a skin rash, or both. Juvenile DM (JDM) may present similarly or as an acute or subacute febrile illness followed by skin, muscle, or sometimes multisystem involvement. The mechanism of disease is associated with induction of the type I interferon pathway.4

The skin involvement in DM may have diverse manifesta- tions, including a heliotrope rash (purplish discoloration) on the Figure 3. Muscle pathology of second biopsy, hematoxylin and eosin eyelids; an erythematosus rash on the face, neck, and anterior staining, Case Two. Inset shows enlargement of the central myofiber. chest (“V-sign”), upper back (“shawl sign”), elbows, or knees; a Used with permission from the author. purplish scaly papular rash on the extensor surface of the hands (Gottron’s papules); thickened and cracked skin on the dorsal and CASE DISCUSSIONS ventral surfaces of the hands (“mechanic’s hands”); and other changes. Subcutaneous calcinosis is a significant problem in JDM Case One and uncommon in adult DM. Cutaneous symptoms in DM have a high impact on lowering quality of life in patients and include This patient developed proximal weakness along with a skin rash, prominent pruritus.7,8 immediately raising the possibility of dermatomyositis (DM). The description of the rash including purplish papules over the The pattern of proximal limb weakness in DM is not distinc- dorsal finger joints suggests the presence of Gottron’s papules. tive and does not distinguish DM from many other myopathies. The normal serum CK makes unlikely certain inherited muscle Significant muscle asymmetries or prominent distal (forearm or diseases but does not diminish the possibility of DM, as CKs are lower leg) weakness together with skin rash should prompt con- often normal in DM. The muscle biopsy demonstrates perivas- sideration for sarcoidosis, for which clinical involvement similar cular and perimysial location of inflammatory cells (Figure 1A) to DM has been recognized.9 Normal serum CK may be present and the presence of perifascicular atrophy (Figures 1A and 1B) in patients with progressing disease and does not exclude the that is evident as basophilic atrophic fibers around the periphery diagnosis. When elevated serum CK is present in DM, reductions of fascicles. Perifascicular atrophy is virtually diagnostic of DM. generally occur with treatment, and elevation with relapse.

Case Two Additional evaluation of adult patients with DM should be performed because of its association with two other important This patient developed slowly progressive weakness of the hands clinical syndromes: interstitial lung disease and malignancy. and quadriceps by history and by physical examination had Pulmonary function tests, chest computed tomography (CT), and marked weakness of finger flexors in contrast to finger extensors. laboratory testing for the presence of antihistidyl transfer RNA This pattern immediately suggests sporadic inclusion body myo- antibodies (anti-Jo-1 antibodies) should be considered in all sitis (sIBM). The muscle biopsy (Figure 3) shows endomysial patients with DM. Malignancy has been estimated to be associ- inflammation with invasion of non-necrotic muscle fibers (arrow) ated with 6-45% of adult patients with DM, with age-associated and rimmed vacuoles (arrowheads). The lack of clinical response increased risk particularly in women over 40 years old. A malig- to prednisone, despite reduction in serum CK values, is charac- nancy evaluation—including physical examination (skin exami- teristic, as is the initial diagnosis as PM. nation, breast and pelvic examinations in women, and testicular and prostate examination in men), blood studies (complete blood GENERAL DISCUSSION count, liver function tests, lactate dehydrogenase, and prostate specific antigen), stool studies for occult blood, CT (chest, The inflammatory myopathies are diseases in which muscle abdomen, and pelvis), and colonoscopy—should be considered appears to be injured by the immune system. The clinical features in every adult patient with a new diagnosis of DM. and pathogenesis have been recently reviewed and much of the material that follows is adapted from such recent reviews.1-5 Muscle biopsy is an important diagnostic procedure in DM. The clinical syndrome in patients with typical skin and muscle The principal subtypes are DM, inclusion body myositis (IBM), features is quite specific for DM though some patients with and PM, though many patients have syndromes that are not easily sarcoidosis have been reported with similar clinical but distinct classified and may best be labeled as nonspecific (or unspecified) pathological features.10-12 The most supportive diagnostic feature myositis. Other subtypes include necrotizing myopathy, overlap of muscle pathology for DM evident in routine clinical studies is syndromes (inflammatory myopathy occurring in a patient with the presence of perifascicular atrophy, small myofibers that are 16 PRINCIPLES OF NCS AND NEEDLE EMG slightly darker and bluish in color in hematoxylin and eosin sec- Polymyositis tions, typically located at the edges of fascicles. Patients with acquired myopathies whose weakness improves Inclusion Body Myositis with immunosuppressive therapies and relapses with taper of such therapy, but lack the rash and pathological features of DM Inclusion body myositis affects adults during middle age and later are challenging to classify. Depending on various criteria, such life. The name was first applied to a patient with symptom onset patients may be categorized as having PM, nonspecific myositis, at age 18 and findings at age 26 consisting of lordotic posture, necrotizing myopathy, overlap syndromes, or other diagnoses. leg limb-girdle weakness, and no atrophy or weakness of the quadriceps. This patient would not meet current criteria for the Patients with subacute progressive symmetrical proximal arm and diagnosis of IBM.13 Although onset over age 50 has been em- leg weakness, without skin rash, and with muscle biopsy features phasized, symptom onset prior to age 50 is common (18-20% of of prominent inflammatory cells surrounding many muscle fibers, patients).14,15 Diagnosis has historically been frequently delayed without perifascicular atrophy, are the patients that are most by a mean of 5-8 years from symptom onset.14,16-19 The mecha- appropriately classified as having PM or nonspecific myositis. nism of disease may be linked to nuclear degeneration and the Various research diagnostic criteria have been considered with redistribution of the normally nuclear protein TDP-43 to the regard to the challenges of PM diagnosis.24-26 The practical issues sarcoplasm.20,21 are avoiding misclassification of certain muscular dystrophies, particularly limb-girdle muscular dystrophy (LGMD), and IBM The clinical presentation of IBM is quite distinct from that of as PM. Most patients with LGMD and IBM meet widely-used other inflammatory myopathies. Atrophy and weakness of wrist criteria for the diagnosis of PM.27 and finger flexors and quadriceps are distinctive and physical examination should focus on careful testing of these muscle As with DM, there is an association with interstitial lung disease, groups. Comparison of wrist and finger extensors with corre- but not a well-established association with malignancy. Serum sponding flexors may demonstrate the greater involvement of CK is almost always elevated in patients with progressing PM. the flexors and asymmetries. Relative preservation of deltoids, Connective tissue diseases should be considered through clinical in comparison to the forearm flexors, can be impressive, in evaluation and antinuclear antibody testing. marked contrast to the pattern of weakness seen in DM and PM. Contrasts between severe biceps weakness, but better preserved The pathological diagnosis of PM is reasonable when there are brachioradialis, and severe deep finger flexor weakness, but -un abundant inflammatory cells surrounding multiple myofibers and commonly involved adductor pollicis, have been emphasized as an absence of rimmed vacuoles. There are considerable differ- well.15 Involvement of tibialis anterior may also be distinctive in ences in opinion, ranging from support for diagnostic criteria that IBM. Dysphagia can be a significant problem, with a prevalence allow a diagnosis of definite PM with biopsy features that include estimated as high as 66%.15 some unspecified combination of muscle degeneration, regenera- tion, necrosis, and inflammatory cells,28 to positions that the di- Serum CK is only modestly elevated; research criteria have pro- agnosis should require invasion by CD8+ T cells (which include posed diagnostic criteria of an upper limit of 12 times the upper cytotoxic and suppressor T cells) of non-necrotic muscle fibers limit of normal,22 though patients with higher values, up to 16 with visible expression of major histocompatibility class 1 by times the upper limit of normal, have been reported.15 Serum immunohistochemistry on these fibers’ sarcolemma. The former electrophoresis and the more sensitive immunofixation should be criteria may lead to misdiagnosis of genetically determined considered because some patients have a detectable serum mono- myopathies, which may have variable degrees of inflammation clonal immunoglobulin population. present (the association of calpain mutations with eosinophilic myositis is one excellent example of this problem29), and IBM as The presence of multiple myofibers surrounded by inflamma- PM, while the latter is restrictive enough that many patients with tory cells and many myofibers with rimmed vacuoles is highly immune-mediated myopathies need an alternative diagnosis, such supportive of a pathological diagnosis of IBM. Both IBM and as non-specific or unspecified myositis.26 PM (see below) may have similar patterns with respect to the location of inflammatory cells as seen in routine studies. The Pathogenesis of Dermatomyositis pattern of inflammatory cells deep within fascicles surrounding and sometimes invading myofibers is distinct from that of DM. The pathophysiology of DM appears to be associated with the What distinguishes IBM from PM in light microscopic examina- family of type I IFN cytokines.4,6 Microarray studies of muscle tion is a sufficient number of rimmed vacuoles, though diagnostic biopsy specimens generating over 2 million data points assay- and research criteria for what constitutes sufficient numbers of ing gene expression of tens of thousands of genes in more than rimmed vacuoles have not been established. The presence of cy- 100 muscle biopsy samples has found the marked overproduc- tomembranous whorls and filamentous inclusions with electron tion of type I interferon (IFN)-inducible transcripts and proteins microscopy are also highly supportive of a diagnosis of IBM. in muscle to be remarkably unique to DM in comparison to all Difficulties with diagnosis occur in patients with typical clinical other muscle diseases studied.4 Elevation of type I IFN-inducible features but few inflammatory cells or with few rimmed vacuoles. transcripts has remarkable specificity for DM. For example, the Small numbers of rimmed vacuoles may be seen in patients with transcript for the type I IFN-stimulated gene 15 (ISG15) was steroid-responsive PM syndromes.23 higher in muscle in all 28 biopsies from adults with DM and

17 INFLAMMATORY MYOPATHIES perifascicular atrophy and children with JDM than in every one 0% (0/25) of other autoimmune myopathy samples, and 0% of 199 non-DM biopsy samples from a wide range of neuromus- (0/15) of normal samples.34 The identity of this protein has not cular diseases. been established.

Some type I IFN-inducible proteins also are highly specific bio- Nuclear Degeneration markers of DM muscle. MxA, for example, is impressively and uniquely (in comparison to other muscle diseases) abundant in Rimmed vacuoles in IBM muscle derive from nuclei. They DM myofibers with perifascicular atrophy and in DM capillaries. are lined with the nuclear membrane proteins lamin A/C and An ubiquitin-like modifier, ISG15 is conjugated to many other emerin.21 Lining of these vacuoles with histone H1 and with emerin proteins in DM muscle. In vitro models of human skeletal muscle was reported in 200835 and with histone 2AX and DNA repair cell cultures, exposed to IFN-α or IFN-β, produces a similar regulatory components (DNA-PK, Hu70, and Hu80) in 2011.36 picture of ISG15 conjugation present in human DM samples. Inclusion body myositis muscle is also notable for the presence Which specific type I IFN might be driving the marked typeI of an aberrantly-localized sacroplasmic protein with the capac- IFN-inducible response in DM muscle is uncertain but recent ity to nonspecifically bind nucleic acids. The nucleic acid-binding evidence suggests that it is interferon-®.30 protein TDP-43 has been identified in IBM non-nuclear -sarco plasm.20,37 TDP-43 is a predominantly nuclear heterogeneous Pathogenesis of Inclusion Body Myositis nuclear ribonucleoprotein that undergoes nucleocytoplasmic shut- tling and associates with translation machinery in the cytoplasm. There are many theories of the pathogenesis of IBM. Toxicity theo- ries of various molecules have dominated the field. Mitochondrial REFERENCES pathology and infectious causes have also been considered. Other considerations have included a potential infectious origin and a 1. Greenberg SA. Theories of the pathogenesis of inclusion body myosi- mitochondrial disorder. Autoimmunity and nuclear degeneration tis. Curr Rheumatol Rep 2010;12:221-228. are perhaps the strongest theories. 2. Greenberg SA. Inclusion body myositis. Curr Opin Rheumatol 2011;23:574-578. 3. Greenberg SA. Pathogenesis and therapy of inclusion body myositis. Autoimmunity Curr Opin Neurol 2012; in press. 4. Greenberg SA. Type 1 interferons and myositis. Arth Res Ther The adaptive immune system is an arm of the immune system 2010;12(Suppl 1):S4. capable of generating highly specific molecular targeting. 5. Greenberg SA. American Association of Neuromuscular and Inclusion body myositis has a highly refined adaptive immune Electrodiagnostic Medicine Course: Inflammatory myopathies. response. Both T cells and B cells undergo affinity maturation, Rochester, MN: American Association of Neuromuscular and selection, and clonal expansion through high-affinity interactions Electrodiagnostic Medicine; 2010. of their receptors (the T cell antigen receptor and B cell surface 6. Greenberg SA. Proposed models of the inflammatory myopathies and immunoglobulin receptor) with target antigen. their potential therapeutic implications. Neurology 2007;69:2008- 2019. Cytotoxic T cell invasion of myofibers has been emphasized 7. Shirani Z, Kucenic MJ, Carroll CL, et al. Pruritus in adult dermatomyo- since the mid-1980s. Molecular analyses of the T cell receptor sitis. Clin Exp Dermatol 2004;29:273-276. 8. Hundley JL, Carroll CL, Lang W, et al. Cutaneous symptoms of der- (analysis of the nucleotide sequences of the variable regions of matomyositis significantly impact patients’ quality of life. J Am Acad their α and β chains) have demonstrated clonal restriction (a Dermatol 2006;54:217-220. limited population of distinct T cell receptor TCR transcript se- 9. Kubis N, Woimant F, Polivka M, et al. A case of dermatomyositis and quences), providing circumstantial evidence of clonal expansion muscle sarcoidosis in a Caucasian patient. J Neurol 1998;245:50-52. (that T cells have been activated by antigen and proliferated). In 10. von Wussow P, Jakschies D, Hartung K, Deicher H. Presence of inter- IBM muscle, antigen-stimulated T cells develop highly specific feron and anti-interferon in patients with systemic lupus erythemato- antigen-directed receptors during poliferation. sus. Rheumatol Int 1988;8:225-230. 11. Lipton JH, McLeod BD, Brownell AK. Dermatomyositis and granu- Inclusion body myositis also has a highly refined B cell response lomatous myopathy associated with sarcoidosis. Can J Neurol Sci with plasma cells31 with immunoglobulin gene rearrangements, 1988;15:426-429. characteristic of clonal expansion in response to local antigen 12. Brateanu AC, Caracioni A, Smith HR. Sarcoidosis and dermatomyosi- tis in a patient with hemoglobin SC. A case report and literature review. stimulation,32 and a permissive environment for ectopic lymphoid 33 Sarcoidosis Vasc Diffuse Lung Dis 2000;17:190-193. structures suggestive of local maturation of B cells in muscle. 13. Yunis EJ, Samaha FJ. Inclusion body myositis. Lab Invest 1971;25:240- The differentiated B cells develop into clonally expanded, highly- 248. refined antigen directed plasma cells that produce and secrete 14. Lotz BP, Engel AG, Nishino H, et al. Inclusion body myositis. immunoglobulins within IBM muscle. Observations in 40 patients. Brain 1989;112(Pt 3):727-747. 15. Badrising UA, Maat-Schieman ML, van Houwelingen JC, et al. Recognition that a B cell-specific response was present in IBM Inclusion body myositis clinical features and clinical course of the muscle and characterization of this response suggested that a disease in 64 patients. J Neurol 2005;252(12):1448-1454. search for circulating IBM autoantibodies might be fruitful. In 16. Badrising UA, Maat-Schieman M, van Duinen SG, et al. Epidemiology 2011, an autoantibody to a human muscle protein of approxi- of inclusion body myositis in the Netherlands: a nationwide study. mately 43 kDa was identified in 52% (13/25) of IBM samples, Neurology 2000;55:1385-1387. 18 NEUROMUSCULAR UPDATE II 17. Beyenburg S, Zierz S, Jerusalem F. Inclusion body myositis: clin- 28. Kamme F, Salunga R, Yu J, et al. Single-cell microarray analysis in ical and histopathological features of 36 patients. Clin Investig hippocampus CA1: demonstration and validation of cellular heteroge- 1993;71:351-361. neity. J Neurosci 2003;23:3607-3615. 18. Lindberg C, Persson LI, Bjorkander J, Oldfors A. Inclusion body myo- 29. Krahn M, Lopez de Munain A, Streichenberger N, et al. CAPN3 mu- sitis: clinical, morphological, physiological and laboratory findings in tations in patients with idiopathic eosinophilic myositis. Ann Neurol 18 cases. Acta Neurol Scand 1994;89:123-131. 2006;59:905-911. 19. Sayers ME, Chou SM, Calabrese LH. Inclusion body myositis: analy- 30. Liao AP, Salajegheh M, Nazareno R, et al. Interferon beta is associated sis of 32 cases. J Rheumatol 1992;19:1385-1389. with type 1 interferon-inducible gene expression in dermatomyositis. 20. Salajegheh M, Pinkus JL, Taylor JP, et al. Sarcoplasmic redistribu- Ann Rheum Dis 2011;70:831-836. tion of nuclear TDP-43 in inclusion body myositis. Muscle Nerve. 31. Greenberg SA, Bradshaw EM, Pinkus JL, et al. Plasma cells in 2009;40:19-31 muscle in inclusion body myositis and polymyositis. Neurology 21. Greenberg SA, Pinkus JL, Amato AA. Nuclear membrane proteins are 2005;65:1782-1787. present within rimmed vacuoles in inclusion-body myositis. Muscle 32. Bradshaw EM, Orihuela A, McArdel SL et al. A local antigen-driven Nerve 2006;34:406-416. humoral response is present in the inflammatory myopathies. J 22. Griggs RC, Askanas V, DiMauro S, et al. Inclusion body myositis and Immunol. 2007;178:547-556 myopathies. Ann Neurol 1995;38:705-713. 33. Salajegheh M, Pinkus JL, Amato AA, et al. Permissive environment for 23. van der Meulen MF, Hoogendijk JE, Moons KG, et al. Rimmed vacu- B-cell maturation in myositis muscle in the absence of B-cell follicles. oles and the added value of SMI-31 staining in diagnosing sporadic Muscle Nerve 2010;42:576-583. inclusion body myositis. Neuromuscul Disord 2001;11:447-451. 34. Salajegheh M, Lam T, Greenberg SA. Autoantibodies against a 43 KDa 24. Hoogendijk JE, Amato AA, Lecky BR, et al. 119th European Neuro muscle protein in inclusion body myositis. PLoS One. 2011;6:e20266. Muscular Centre international workshop: trial design in adult idio- 35. Nakano S, Shinde A, Fujita K, et al. Histone H1 is released from myo- pathic inflammatory myopathies, with the exception of inclusion nuclei and present in rimmed vacuoles with DNA in inclusion body body myositis, 10-12 October 2003, Naarden, The Netherlands. myositis. Neuromuscul Disord 2008;18:27-33. Neuromuscul Disord 2004;14:337-345. 36. Nishii M, Nakano S, Nakamura S, et al. Myonuclear breakdown in 25. Amato AA, Griggs RC. Unicorns, dragons, polymyositis, and other sporadic inclusion body myositis is accompanied by DNA double mythological beasts. Neurology 2003;61:288-289. strand breaks. Neuromuscul Disord 2011;21:345-352. 26. van der Meulen MF, Bronner IM, Hoogendijk JE, et al. Polymyositis: 37. Weihl CC, Temiz P, Miller SE, et al. TDP-43 accumulation in inclu- an overdiagnosed entity. Neurology 2003;61:316-321. sion body myopathy muscle suggests a common pathogenic mecha- 27. Bohan A, Peter JB. Polymyositis and dermatomyositis (first of two nism with frontotemporal dementia. J Neurol Neurosurg Psychiatry parts). N Engl J Med 1975;292:344-347. 2008;79:1186-1189.

19 20 Myotonic Dystrophies

Bassam A. Bassam, MD Professor of Neurology Director, Neuromuscular and EMG Laboratory University of South Alabama Mobile, Alabama

CASE PRESENTATION no diplopia or delayed eye opening after forceful closure. He was noted to have mild atrophy of the glutei and sternocleidomastoid History muscles bilaterally. He had difficulty rising from a sitting position without pressing on the chair arms. Muscle strength examination A 58-year-old man presented with a 10-year history of slowly using the Medical Research Council rating scale revealed weak- increasing muscle weakness. He had difficulty standing from a ness of neck flexors 4/5, shoulder abductors 4+/5, biceps 4+/5, sitting or squatting position, climbing stairs, and getting in his truck long finger flexors 4+/5, hip flexors 4/5, hip abductors 4+/5 bilat- as well as mild hand grip, arm, and neck weakness. He had no erally, and ankle extensors 4+/5 and normal strength for all other speech, swallowing, or breathing difficulty. He denied any muscle remaining muscle groups. No muscle hypertrophy, fasciculations, stiffness, spasm, or twitches, except for having infrequent episodes stiffness, or hand grip myotonia were found; however, he was of deep ache pain in the thighs, and less frequently in the shoulders noted to have mild thenar muscle percussion myotonia. Tendon and upper arms. The patient was seen by a local neurologist 2 reflexes were symmetric rated 2+, and no pathological reflexes years earlier, and reportedly his needle electromyography (EMG) were elicited. Sensory testing, coordination, and gait examination showed a mix of positive waves and myopathic motor unit action were all normal. potentials (MUAPs). He had declined a muscle biopsy. Initial Differential Diagnosis The patient’s past medical history included diabetes mellitus, treated with metformin, bilateral cataracts removed 5 years prior, The patient presented with chronic, slowly progressive proxi- and an appendectomy at age 34 years. He smokes half a pack of mal muscle weakness of the shoulder, hip girdle, and neck. tobacco per day and drinks two to three beers daily. The family The differential diagnosis in patients with proximal weakness medical history is positive for diabetes and his father died at age should include muscular dystrophies; metabolic, congenital, 61 years secondary to heart disease. He also was reported to have or mitochondrial myopathies; and acquired myopathies, such had muscle weakness. The patient had a living paternal aunt di- as inflammatory, toxic, or endocrine. Other diagnoses, such as agnosed with muscular dystrophy. late-onset spinal muscular atrophy (SMA) and generalized my- asthenia gravis should also be considered. Motor neuron disease Examination usually presents in a different anatomical distribution of muscle weakness, and it is not likely in this case given that the patient’s The patient’s general physical examination was unremarkable. weakness remained mild after 10 years. An inherited autosomal The neurological examination revealed normal cognitive func- dominant myopathy should be suspected, given that the patient’s tioning. Cranial nerves II-XII were normal on examination, father reportedly had muscle weakness and he has a living pater- except for mild ptosis and subtle bilateral facial weakness. He had nal aunt diagnosed with muscular dystrophy. 21 MYOTONIC DYSTROPHIES The predominantly proximal weakness suggests a diagnosis of DM2. The diagnosis of DM2 was confirmed by a positive genetic muscular dystrophy (such as limb girdle muscular dystrophy), test for expanded tetranucleide repeats mutation in the zing finger metabolic myopathy (such as adult onset acid maltase deficiency), protein 9 (ZNF9) on chromosome 3. myotonic dystrophy (or dystrophia myotonica) (DM), late onset SMA, or mitochondrial myopathy. Although adult onset acid DISCUSSION maltase deficiency presents with a slowly progressive weakness over many years, it is usually associated with respiratory muscles Myotonic Dystrophy weakness and with no clinical myotonia observed. Mitochondrial myopathy is a relatively static proximal weakness with poor The myotonic disorders are a heterogeneous entity, generally endurance and can be associated with external ophthalmoplegia. divided into dystrophic and nondystrophic groups in which myo- It has a different inheritance pattern. Late onset SMA usually tonia might be the single common clinical feature. The nondys- presents at an earlier age than this case, with insidious weakness trophic myotonias can be further subdivided based on the type of and atrophy of the pelvic girdle and proximal leg muscles. It is muscle channel involved and the presenting features. Myotonic followed by involvement of the shoulder girdle and upper arms dystrophy is overwhelmingly the most common disease among muscles, associated with fasciculations and depressed tendon the myotonic disorders and it is considered the most common reflexes. Muscular dystrophy, especially DM, should be highly form of muscular dystrophy in adults. The disease was first considered given that the patient has muscle weakness, diabetes, recognized by Steinert in 1909. It is inherited in an autosomal bilateral cataract, and percussion myotonia as well as a positive dominant manner of variable penetrance, anticipation, and a ma- family history for weakness, diabetes, and heart disease. The ternal transmission in the congenital form.1,2 Although classified classical myotonic muscular dystrophy type 1 (DM1) is unlikely as a muscle disease with predominant myotonia and progressive with predominantly proximal weakness. However, the patient’s weakness, DM is clearly a disease of multisystemic nature owing proximal weakness, percussion myotonia, associated systemic to variable ocular, cardiac, endocrine, reproductive, gastrointesti- diseases, and family history are consistent with an autosomal nal, skin, and cognitive involvement. The disease is classified into dominant inheritance and are suggestive of myotonic muscular DM1 and DM2. There may be more genetically distinct forms yet dystrophy type 2 (DM2), also called proximal myotonic myopa- to be identified. thy (PROMM). Myasthenia gravis is not highly suspected given that the patient has no diplopia, ptosis bulbar weakness, increased Myotonic Dystrophy Type 1 fatigue, or weakness fluctuations. Myotonic dystrophy type 1 is the most common form of muscu- Laboratory Tests and Electrodiagnostic lar dystrophy, with an incidence of 13.5/100,000 based on Swiss Studies data, and an estimated prevalence of around 5/100,000 for most European and North American populations.3 It is the most severe Normal or negative laboratory studies included: blood count and form among the myotonic disorders. Great variability of clini- differential, serum chemistry, thyroid studies, liver function tests, cal expression exists, ranging from the severe congenital form sedimentation rate, serum lactate, acetylcholine receptor antibod- to subclinical cases. Although DM1 can present at any age, in ies, and hemoglobin A1C. The serum creatine kinase (CK) was the majority of cases onset of symptoms and diagnosis occurs 642 IU/L (normal < 210 IU/L). in the late teens or early adulthood. The most common present- ing complaint is usually muscle weakness. Myotonia is often a “minor nuisance” compared with other symptoms. Less com- monly, patients with DM1 may initially present with subnormal intelligence; premature cataracts; and pulmonary, endocrine, or Figure. Waxing and waning discharges in the form of positive waves (100 gastrointestinal symptoms. Neck flexor and distal limb muscle ______µV/20 ms). weakness usually starts early in the disease course, especially in the forearm and finger flexors, and in the ankle dorsiflexors. It then slowly progresses to affect the proximal muscles, often asso- Electrodiagnostic (EDX) studies were performed. A standard ciated with muscle atrophy.4 Facial features including mild ptosis, motor and sensory nerve conduction study and an F wave were weakness of the eyes and mouth closure, frontal balding, a long all normal. Repetitive motor nerve stimulation of the ulnar and face with hollowed cheeks, and temporalis and masseter wasting spinal accessory nerve at 3 Hz at rest and post-exhaustion exer- are other characteristics. Action myotonia, such as delayed re- cise showed no neuromuscular junction abnormality. The needle laxation of the fingers following a forceful hand grip, is a minor EMG examination demonstrated runs of discharges with waxing complaint, aggravated by cold weather. It will typically diminish and waning amplitude and frequency, in the form of positive with repeated attempts (“warm-up” phenomenon). Often patients waves more prominent in the distal muscles (Figure). In addition, deny having myotonia or appear to be unaware of it and it has the needle EMG demonstrated fibrillation potentials and frequent to be actively elicited by percussion of the thenar eminence, the polyphasic MUAPs. Many of these were of short duration and tongue, or during a needle EMG. small amplitude with early recruitment predominantly in the proximal limb muscles. Overall the EDX study was characteristic The extramuscular manifestations in DM patients include a of myotonia with associated myopathic features predominantly number of frequent multisystemic clinical features. Posterior in the proximal muscles, supportive of the clinical diagnosis of subcapsular cataracts of variable severity and lenticular opaci-

22 PRINCIPLES OF NCS AND NEEDLE EMG ties are invariably present in patients over the age of 20 years. mortality in affected infants are likely bronchial aspiration and These are best identified by slit lamp examination.5 Cardiac con- neonatal respiratory distress due to diaphragmatic weakness and duction defects are common and seen in up to 90% of patients. hypoplasia and intercostal muscle weakness.21 Talipes may occur Cardiomyopathy is rare and the cardiac abnormalities do not cor- in about half the cases and generalized arthrogryposis is reported relate with the severity of muscle dysfunction.6 With prolonged in a small portion of patients.22 Myotonia is absent in infancy disease duration, fatal arrhythmias and sudden death may occur in and becomes evident later in childhood. Surviving infants show DM1 patients.7 Insulin insensitivity, hyperglycemia, and hyper- delayed motor and speech development, swallowing difficulty, insulinemia following a glucose tolerance test are common. The and mild-to-moderate cognitive deficits. Once adolescence is incidence of overt diabetes mellitus is not increased.8 Testicular attained, the disease follows the same course as the later onset failure, low serum testosterone, and impotence in males are disease. The characteristic facial features and eliciting myotonia seen in both DM1 and DM2.9 Cognitive and neuropsychological in the mother allow instant recognition of the diagnosis in many testing in DM frequently reveals mild cognitive deficits, poor cases. Although the prenatal diagnosis of DM is readily accom- insight, and behavioral and personality disorders.10 Magnetic plished by testing for cytosine-thymine-guanine (CTG) repeats in resonance imaging nonspecific abnormalities of cerebral white the amniotic fluid or chorionic villi, it is not possible to predict matter and abnormal positron emission tomography have been whether a fetus with an expanded mutation will have either con- reported.11 Smooth muscle involvement of the gastrointestinal genital or later onset DM. tract may cause dysphagia, decreased motility, constipation, and fecal incontinence.12 Diaphragmatic dysfunction may account for Genetics and Pathogenesis the alveolar hypoventilation, frequently seen in DM patients. It is more striking in infants with the congenital form. Sleep apnea, Myotonic dystrophy type 1 is caused by an expansion of CTG tri- hypersomnia, and disrupted sleep are also common.13 nucleotide repeats in the myotonin protein kinase (DMPK) gene on chromosome 19q13.2.23 The number of the CTG repeats may Myotonic Dystrophy Type 2 (or Proximal expand with the next generation due to anticipation phenomena Myotonic Myopathy) and correlates with the muscle dysfunction severity.24 Myotonic dystrophy type 2 is caused by an expansion of a CCTG tetranu- Myotonic dystrophy type 2 was first identified in 1994 as an au- cleotide repeat within intron 1 of the ZNF9 gene on chromosome tosomal dominant disease, similar to DM1 in most of the clinical 3. This very large repeat accumulates as RNA foci in the nuclei features, including muscle weakness, myotonia, and multisystem of the affected tissue.25 Muscle weakness and the multisystemic involvement. However, it has a characteristic pattern of muscle clinical features common to both DM1 and DM2 are caused by weakness and is genetically distinct from DM1.14,15 The incidence the toxic gain effect on cells subsequent to these repeat expan- of DM2 is uncertain, but it is seemingly less common than DM1. sions in RNA.26 The age of onset is usually between 20 and 60 years, although it may start at an earlier age. Anticipation is much milder than Diagnosis in DM1, and congenital forms have not been described.16 The disease may present with intermittent disabling pain and stiffness Myotonic dystrophy diagnosis can be achieved on clinical in the thighs, shoulders, or upper arms. The weakness is predomi- grounds and with needle EMG. Genetic testing confirms typical nantly proximal and progresses to involve the neck flexors, long or atypical presentation and is commercially available for both finger flexors, elbow extensors, hip flexors, and knees extensors. DM1 and DM2. Minors should not be tested unless symptomatic The main complaint of most affected patients is hip girdle and or a diagnosis is necessary. Prenatal testing via amniocentesis proximal lower limbs weakness. The weakness may fluctuate or chorionic villous sampling assesses fetal risk in diagnosed and/or worsen during pregnancy. Clinical myotonia is variably parents. Serum CK may be normal or mildly elevated. Standard present and may be missed. It is associated with “warm-up” phe- motor and sensory conduction studies are normal. Needle EMG nomena. In a case report it was evident only during pregnancy. demonstrates myotonic discharges often elicited by needle However, in a large series electrical myotonia was demonstrable movements or voluntary contraction. Although at times several in the majority of patients.17,18 Cognitive and systemic abnormali- muscles may need to be tested before discharges can be found, ties are generally milder than in DM1, and fatal cardiac arrhyth- they are less abundant in DM2. Diffuse fibrillations and positive mias or sudden death rarely occur. waves, often masked by the myotonic discharges (as well as myopathic MUAPs and early recruitment), are detected in weak Congenital Myotonic Dystrophy distal and/or proximal limb muscles. Electrical myotonia may not be elicitable in infants with the congenital form of DM, but it Congenital myotonic dystrophy was the first myotonic disorder to can be easily demonstrable in the mother’s hand muscles. Muscle be recognized. It is a distinctive and potentially fatal form of DM, biopsy, which is not required for diagnosis, shows characteristic with the affected parent always being the mother. Hydramnios type I fiber atrophy and occasional ring-band fibers in DM1. and reduced fetal movements in the later part of pregnancy are Type II fiber atrophy and pyknotic nuclei are common in DM2 common. The principal clinical features of the disease are rec- with abundant central nuclei and intranuclear RNA inclusions. ognized after birth. It is unusual for all features to be present in Necrotic muscle fibers are uncommon findings and increased a single patient. Major clinical features include: bilateral facial connective tissue is usually mild. This is mostly seen in the late weakness in over 85% of cases, accompanied by a “tented” upper stages of the disease.27 lip, prominent jaw muscle weakness, hypotonia, and feeding and sucking difficulty of varying severity.19,20 The primary causes of 23 MYOTONIC DYSTROPHIES Therapy and Prognosis Myotonic dystrophy type 2 has rarely been associated with fatal arrhythmias or respiratory insufficiency, and the congenital form There is no known effective treatment for muscle weakness or has not been identified in DM2 cases. curative therapy for the multisystemic manifestations in DMs. Small clinical trials of creatine monohydrate and recombinant REFERENCES human growth hormone and controlled trials of dehydroepi- androsterone sulfate in DM1 patients showed no efficacy.28 1. Steinert H. Myopathologische beitrage 1, Uber das klinische und Therapies are primarily focused on drug treatment of myotonia anatomische bild des muskelschwundes der myotoniker. Dtsch Z and muscle pain, along with management of the various systemic Nervenhlk 1909;37:58-104. manifestations. Patients with DM rarely complain of myotonia, 2. Magee AC, Hughes AE. Segregation distortion in myotonic dystrophy. and drug management should be limited to those with bother- J Med Genet 1998;35:1045-1046. 3. Emery AEH. Population frequencies of inherited neuromuscular dis- some myotonia due to the potential cardiac conduction side eases, a world survey. Neuromuscul Disord 1991;1:19-29. effects. Quinine sulfate 0.5 g twice daily, procainamide 0.5 three 4. Thornton C. the myotonic dystrophies. Semin Neurol 1999;20:25-33. times daily, or tocainide 400 mg three times daily are effective 5. Burian HM, Burns CA. Ocular changes in myotonic dystrophy. Am J treatments for myotonia, but they potentiate cardiac arrhythmias. Ophthalmol 1967;63:22. None of these drugs should be used in the presence of any cardiac 6. Phillips MF, Harper PS. Cardiac disease in myotonic dystrophy. disturbances. Mexiletine 200 mg three times daily, with electro- Cardiovasc Res 1997;33:13-22. cardiogram (EKG) monitoring, or phenytoin 100 mg three times 7. Sabovic M, Medica I, Logar N, et al. Relation of CTG expansion and daily are helpful in relieving disabling myotonia.29 Muscle pain clinical variables to electrocardiogram conduction abnormalities and is usually secondary to weakness and deconditioning and can be sudden death in patients with myotonic dystrophy. Neuromuscular treated with nonsteroidal anti-inflammatory drugs or other ap- Disord 2003;13(10):822-826. propriate pain medications. 8. Moxley RT, Griggs RC, Goldblatt D. decreased insulin sensitivity of forearm muscles in myotonic dystrophy. J Clin Invest 1978;62:857-867. 9. Day JW, Roelofs R, Leroy B, et al. Clinical and genetic characteristics General management includes careful cardiac monitoring for of a five-generation family with a novel form of myotonic dystrophy; conduction disturbances including screening with yearly EKGs. DM2. Neuromuscl Disord 1999;9:19-27. Cardiology referral, Holter monitoring, and echocardiograms are 10. Delaporate C. Personality patterns in patients with myotonic dystrophy. necessary if EKGs show conduction defect or arrhythmias. In Arch Neurol 1998;55:635-640. patients with second or third degree heart block or in those for 11. Hund E, Jansen O, Koch MC, et al. Proximal myotonic myopathy with whom there is concern of unpredictable sudden death, pacemaker MRI white matter abnormalities of the brain. Neurology 1997;48:33-37. placement might be required. Pulmonary function tests and pre- 12. Lecointe-Besancon I, Leroy F, Devroed G, et al. A comparative study cautions to avoid respiratory problems, including increased risk of esophageal and anorectal motility in myotonic dystrophy. Dig Dis of aspiration and pneumonia, are recommended. Noninvasive 1999;44:1090-1099. ventilatory assistance (bilevel positive airway pressure or contin- 13. Coccagna G, Mantouant M, Parch C, et al. Alveolar hypoventila- tion and hypersomnia in myotonic dystrophy. J Neurol Neurosurg uous positive airway pressure) might be required in patients with Psychiatry 1975;38:977. clinically-significant hypoventilation, diaphragmatic weakness, 14. Ricker K, Koch MC, Lehhmann-Horn F, et al. Proximal myotonic and sleep apnea. Ophthalmologic evaluation and cataract surgery myopathy; a new dominant disorder with myotonia, muscle weakness, might be needed. Orthopedic measures might be needed in chil- and cataracts. Neurology 1994;44:1448-1452. dren with talipes and other joint deformities. Orthotic devices for 15. Ricker K, Koch MC, Lehhmann-Horn F, et al. Proximal myotonic my- patients with foot drop help to facilitate their gait. Swallowing opathy, clinical features of a multisystemic disease similar to myotonic assessment in patients with dysphagia is important to avoid risk dystrophy. Arch Neurol 1995;52:25-31. of aspiration, pneumonia, and poor nutrition. Speech therapy in 16. Schneider C, ZieglerA, Ricker K. Proximal myotonic myopathy. patients with dysarthria is helpful. Endocrine status and meta- Evidence for anticipation in families with linkage to chromosome 3q. bolic function assessment in symptomatic patients or those sus- neurology 2000;55:383-388. pected with dysfunction is necessary. Anesthetists and surgeons 17. Newman B, Meola G, O’Donovan DG, et al. Proximal myotonic myopathy (PROMM) presenting as myotonia during pregnancy. should be made aware of the hazards associated with anesthesia Neuromuscul Disord 1999;9:144-149. and surgery in patients with DMs. Nonstrenuous exercise is safe 18. Day JW, Ricker K, Jacobsen JF, et al. Myotonic dystrophy type 2. and may improve physical fitness and endurance. Genetic coun- Molecular, diagnostic, and clinical spectrum. Neurology 2003;60:657-664. seling is essential and of particular value for individuals with an 19. Hageman AT, Gabreels FJ, Liem KD, et al. Congenital myotonic dys- affected parent. It is also essential for those affected individuals trophy; a report on thirteen cases and a review of the literature. J Neurol who wish to have children or to have a prenatal diagnosis. Sci 1993;115:95-101. 20. Harper PS. Congenital myotonic dystrophy in Britain, clinical aspects. Most DM patients remain mobile throughout their life and are Arch Dis Child 1975;50:505. usually free from the contractures seen in most disabling neuro- 21. Bossen EH, Selburne JD, Verkauf BS. Respiratory muscle involvement muscular disorders. It is important to recognize that life expec- in infantile myotonic dystrophy. Arch Pathol 1974:97:250. tancy in DM1 patients is significantly reduced due to increased 22. Dyken PR, Harper PS. Congenital dystrophia myotonica. Neurology 1973;23:465. risk of fatal arrhythmias with sudden death, respiratory disease, 23. Meola G. Genetic and clinical heterogeneity in myotonic dystrophies. neoplasia, and coronary artery disease. Longitudinal and cohort Muscle Nerve 2000;13:1789-1799. studies showed 7.3 times higher mortality rate than the age- matched control subjects with a mean age at death of 53 years.30,31

24 PRINCIPLES OF NCS AND NEEDLE EMG 24. Harper PS, Harley HG, Reardon W, et al. Review article: anticipation in myotonic dystrophy; New light on an old problem. Am J Hum Genet 1992;51:10-16. 25. Liquori CL, Ricker K, Moseley ML, et al. Myotonic dystrophy type 2 caused by a CCTG expansion in intron 1 of ZNF9. Science 2001;293:864-867. 26. Day JW, Ranum LP. RNA pathogenesis of the myotonic dystrophies. Neuromuscl Disord 2005;15(1):5-16. 27. Vihola A, Bassez G, Meola G, et al. Histopathological differences of myotonic dystrophy type 1 (DM1) and PROMM/DM2. Neurology 2003;60(11):1854-1857. 28. Tarnopolsky M, Mahoney D, Thompson T, et al. Creatine monohydrate supplementation does not increase muscle strength, lean body mass, or muscle phosphocreatine in patients with myotonic dystrophy type 1. Muscle Nerve 2004;29(1):51-58. 29. Griggs RC, Davies RJ, Anderson DC, et al. Cardiac conduction in myotonic dystrophy. Am J Med 1975;59:37. 30. de Die-Smulders CE, Howeler CJ, Thijs C, et al. Age and causes of death in adult-onset myotonic dystrophy. Brain 1998;121:1557-1563. 31. Mathieu J, Allard P, Potvin L, et al. A 10-year study of mortality in a cohort of patients with myotonic dystrophy. Neurology 1999;52:1658- 1662.

25 26 NEUROMUSCULAR UPDATE II

Nondystrophic Myotonic Disorders

Bakri Elsheikh, MBBS Associate Professor of Clinical Neurology Director of the EMG Laboratory Wexner Medical Center at the Ohio State University Columbus, Ohio

CASE PRESENTATION muscles atrophy, cataract, or thinning of the neck muscles. History Laboratory Studies

A 60-year-old woman presents for evaluation of muscle stiffness Serum creatine kinase (CK), complete blood count, and pain since age 7. She recalls that during winter in elementary electrolytes, liver function, and thyroid-stimulating hormone school she would not be able to unzip her coat without the are all unremarkable. Electrodiagnostic (EDX) testing reveals teacher’s help. In addition to the cold weather, her stiffness normal sensory and motor nerve conduction studies. Needle is worse with any repeated activity or exercise. She reports electromyography (EMG) shows diffuse myotonic discharges. difficulty shoveling snow from the driveway and pulling weeds Differential Diagnosis in the garden. She states, “I learned to live with it all these years, The differential diagnosis for a patient with muscle stiffness, pain, but lately I feel worse.” She describes fatigue and aching muscle and cold-induced weakness includes: pain affecting her arms and thigh muscles. Her past medical • Nondystrophic myotonia history includes remote history of idiopathic thrombocytopenic • Myotonia congenita (MC) purpura, hypertension, and Hashimoto’s thyroid disease; she had • Paramyotonia congenita (PC) hysterectomy and right carpal tunnel surgery. Her medications • Sodium channel myotonias include hydrochlorothiazide, cozaar, atenolol, levothyroxine, and • Myotonic dystrophy trazodone. She is allergic to sulfa and amoxicillin. She retired as a The patient’s cardinal symptoms are muscle stiffness, pain, flight attendant and moved to Ohio recently. She has no siblings. and cold-induced weakness since childhood. The presence of She reports no similar history in the family, including her son and paradoxical myotonia (worsening with activity), eyelid myotonia, her two grandchildren. prominent upper limb and face involvement, extreme sensitivity to cold, and widespread myotonic discharges on needle EMG Examination makes PC a likely diagnosis (see discussion). Specialized EDX testing (short exercise test) and genetic testing helped establish She is afebrile and blood pressure is normal. Mental status is the diagnosis. normal and language is intact. Her cranial nerves examination is unremarkable except for eyelid myotonia that worsens DISCUSSION with repeated testing. She has normal muscle strength, 2+ and symmetric muscle stretch reflexes, and down going plantar Myotonic disorders have in common a peculiar clinical responses. She has normal pin prick and vibratory sensation. phenomenon of delayed skeletal muscle relaxation following Furthermore, she has hand-grip myotonia that worsens with a trigger such as voluntary muscle contraction or percussion.1 repeated trials. She has no frontal balding, temporalis or masseter 27 NONDYSTROPHIC MYOTONIC DISORDERS Clinical myotonia is associated with needle EMG recordings phenotype. Subsequently, sodium channel mutations were of runs of repetitive positive waves or fibrillation potentials identified in patients with all the clinical phenotypes. representing repetitive muscle fiber action potentials. Typical myotonic discharges demonstrate waxing and waning of both In addition to the primary myotonic disorders described above, amplitude and frequency, producing a peculiar sound that is other disorders with occasional myotonia were described. These compared to a dive bomber or a revving chain saw.2 Patients are primarily myotonic discharges seen on needle EMG without with myotonia usually report stiffness and difficulty relaxing clinical myotonia. Examples include acid maltase deficiency, muscles. Eyelid myotonia can lead to difficulty opening the eyes hypothyroidism, and inflammatory myopathies as well as the causing a squint that may last for minutes.1 Parents might report effects of medications such as colchicine, statins, and clofibrate.2 a child is having difficulty opening the eyes when crying. Lid lag However, it should be noted that in some of these cases, complex phenomenon is noted with downward gaze after sustained upward repetitive discharges might be mistaken for myotonic discharges.5 gaze. The patient will appear as if staring or having a frightened look.1 Stiffness in the legs may cause tripping and falls. Myotonia Myotonia Paramyotonia Sodium channel Disease congenita congenita congenita myotonia On examination, action myotonia is elicited by having the patient forcefully close the eyes or make a tight fist while observing for Potassium Becker’s Thomsen’s Eulenburg’s difficulty relaxing the muscles. This is repeated successively three Eponyms aggrivated disease disease disease to four times. Usually repeated activity will improve the myotonia; myotonias* a phenomenon referred to as “warm-up.” This is in contrast with paramyotonia (paradoxical myotonia) where repeated activity Inheritance AR AD AD AD worsens the myotonia. Gene CLCN1 CLCN1 SCN4A SCN4A

Late Infancy and First decade of Percussion myotonia is evoked by tapping the thenar eminence Age at onset or the posterior forearm (wrist extensor) muscles with a reflex childhood early childhood life hammer. Tongue myotonia is elicited by tapping the tongue lightly Arms > legs ± Arms and face Arms and face > Myotonia Legs > arms between two tongue blades while observing for a contraction face > legs legs resembling a ring.1 Warm-up and Repeated Paradoxical Warm-up Warm-up paradoxical Myotonic disorders are divided into myotonic dystrophies and actvity myotonia nondystrophic myotonias. Myotonic dystrophies were described myotonia Cold Absent or earlier in this course. The term “nondystrophic myotonias” is used Absent or mild Extreme Absent to severe to refer to an inherited group of skeletal muscle channelopathies sensitivity mild characterized by muscle stiffness and EDX evidence of myotonic Cold- or Transient 2 exercise- discharges. In contrast to the myotonic dystrophies, this group weakness with Weakness Rare induced Absent of disorders lack progressive weakness and extramuscular initiation of weakness is movement manifestations (cardiac, ocular, respiratory, gastrointestinal, and common endocrine). Occasionally, differentiating an early dystrophy, Eyelid particularly myotonic dystrophy type 2, from nondystrophic Rare Rare Common Common myotonia myotonia can be challenging.3 Subgroups of nondystrophic myotonia are identified based on the clinical and EDX phenotype, Muscle pain Rare Rare Common Common channel involved, and genetic mutation (Table). The advances in Muscle Common Common Less common Less common the understanding of the molecular and genetic pathogenesis have hypertrophy helped to simplify the approach to patients with these disorders. Mild-to- The clinical phenotype and EDX pattern are useful in determining Normal-to- Normal-to-mild Serum CK moderate Mild elevation mild elevation elevation the channel involved and thus guide the genetic testing. elevation

Myotonic Myotonic Myotonic The nondystrophic myotonias are divided into two major groups Myotonic discharges discharges discharges based on the type of ion channel involved: chloride and sodium Needle EMG discharges less profuse and profuse and profuse and prominent channelopathies. The chloride channelopathies include patients widespread widespread widespread with autosomal recessive (AR) MC (Becker’s disease) and patients with autosomal dominant (AD) MC (Thomsen’s disease). The sodium channelopathies encompass two groups of disorders: Chloride Channelopathies PC and sodium channel myotonias (SCMs). In addition to the muscle stiffness, PC is characterized by concomitant episodes of Myotonia Congenita extreme cold sensitivity and cold-induced weakness. This helps differentiate it from the sodium channel myotonias, a group that Myotonia congenita is the most common skeletal muscle ion shares lack of cold sensitivity and potassium aggravation, hence channelopathy. It is caused by mutation in the CLCN1 gene on the name “potassium aggravated myotonia.” Several subtypes chromosome 7q35, which encodes the skeletal muscle chloride (myotonia fluctuans, myotonia permanens, and acetazolamide channel 1.6 Chloride control resting membrane potential and responsive myotonia) were described based on the clinical impairment of chloride conductance in patients with MC shift 28 NEUROMUSCULAR UPDATE II the membrane potential towards hyperexcitability. There are two of many uncommon pathogenic mutations. Thus, one should be major types of MC: an AD form (Thomsen’s disease) described cautious interpreting commercial testing limited only to common in the 19th century (1876) by Julius Thomsen (who shared the mutations. Also, a recent trial showed 6% of a MC cohort with a disease with multiple members of his family)7 and an AR form single recessive mutation has an exon deletion or duplication.14 (Becker’s disease) which was described decades later (1966) by P.E. Becker.8,9 Contrary to old data showing the AD type to be It is important to counsel patients on the effect of the activity more common, recent studies suggests the reverse, with the AD and cold exposure. Genetic counseling should be offered as type representing around 37% of the mutation-positive persons.4 well. Many patients might not require drug treatment for the Usually, patients will have myotonia and stiffness present from myotonia. However, those with disabling symptoms will benefit birth, however symptoms may not develop until childhood. from therapy. A Cochrane Review concluded a lack of good- Muscle stiffness is enhanced by cold and inactivity and relieved quality data to allow for specific recommendations.15 Many by exercise. Muscle strength is typically normal. Transient specialists consider mexiletine, an anti-arrhythmic drug, as the weakness is described and it usually last few seconds. Muscle drug of choice for treatment of myotonia. It is proven safe and hypertrophy, sometimes pronounced, giving a muscular physique well tolerated in a recent study by the Consortium for Clinical and athletic appearance, is a common feature. Overall, muscle Investigation of Neurologic Channelopathies. The data was pain is underestimated in myotonic disorders; it is reported in presented at the 2012 American Academy of Neurology annual 28% of MC patients.10 Myotonic symptoms tend to improve with meeting showing that mexiletine at a dose of 200 mg three times age, but they do not disappear. daily is effective therapy for symptoms and signs of myotonia in patients with nondystrophic myotonias.16 Because of the drug’s In general, the two types of MC (recessive and dominant) share potential to cause prolonged QT and cardiac arrhythmia, an initial many clinical features but some are different (Table). The AR electrocardiogram and close cardiac monitoring for prolonged QT type is usually more severe, primarily affecting the legs, with are needed. Other side effects include gastrointestinal, dizziness, more frequent muscle hypertrophy and transient weakness with and tremors. Other medications used with variable results are initiation of movement.3 The AD form has an earlier age of onset flecainide, dilantin, and carbamezapine. and affects mainly the arms. Sodium Channelopathies Serum CK is normal or slightly elevated. Nerve conduction study usually shows normal sensory and motor conductions. Needle Paramyotonia Congenita EMG shows profuse and widespread myotonic discharges. Muscle biopsy is not needed and if performed it is usually normal or shows Paramyotonia congenita is an AD disease caused by a mutation in mild nonspecific changes. The short exercise test, a measure of the SCN4A gene on chromosome 17q23, which encodes the alpha muscle membrane excitability, involves comparing compound subunit of the voltage-gated sodium channel.17 It is allelic with muscle action potential (CMAP) response at rest with CMAP hyperkalemic periodic paralysis, hypokalemic periodic paralysis responses following a series of 10 s of exercise. A baseline CMAP type 2, and sodium channel myotonias. Though myotonia is seen is recorded at rest from the abductor digiti minimi (ADM) muscle in 75% of hyperkalemic periodic paralysis patients, the most after which the patient is asked to perform a maximal isometric prominent clinical picture is weakness.18 Described by Eulenberg contraction of the ADM muscle and then relax completely. in the 19th century,19 PC usually manifests in the first decade of Postexercise CMAP recordings are obtained in 10-s intervals for life. The myotonia is more prominent in the arms, face, and tongue. 1 min. The amplitudes of the postexercise responses are compared Muscle hypertrophy is less common compared to patients with to the baseline response.11 A decrease by > 20% is considered MC. Some of the main features that help differentiate PC from abnormal. The test is repeated three times at normal temperature MC are paradoxical myotonia (worsens with repeated activity), and after cooling to help identify various specific patterns.11,12 extreme cold sensitivity, and cold-induced weakness lasting up There are three patterns associated with nondystrophic myotonias, to few hours (Table). Some patients complain of muscle pain, the most common or which in MC patients is pattern II. It shows though it is usually mild. In general, clinical features are helpful in a drop in CMAP amplitude with a rapid recovery in 1 min. There differentiating the various types of nondystrophic myotonias but is less decrement with repeated trials, reflective of the clinical there is significant overlap and thus mutation analysis is required “warm-up” phenomenon. Furthermore, there is no additional to reach a specific diagnosis. effect to cooling. This pattern is more common in the AR MC patients but might be seen in other diseases with chloride channel Serum CK is usually normal or mildly elevated. Needle EMG shows involvement, including AD MC and in myotonic dystrophy myotonic discharges. A recent study described other discharges, patients.3 Some of the AD MC patients manifest pattern III, which including dolphin-sounding low amplitude (100-600 μV), high reflects a normal response or lack of an abnormal pattern. A recent frequency musical 150-250 Hz discharges in large limb muscles.13 study found no additional benefit to performing 10 Hz repetitive Also higher amplitude tornadeo-shaped, neuromyotonia-like nerve stimulation.13 Commercially-available genetic testing helps discharges are noted in some patients.13 The short exercise test establish the diagnosis (www.genetests.org). There are more than shows a gradual and relatively prolonged decrement after brief 120 described chloride channel mutations, the majority of which exercise, indicative of pattern I.11 The decrement increases are associated with AR MC. Furthermore, for reasons that are with repeated trials and with cooling reflective of the clinical not clear, a specific mutation can result in AR MC in one family worsening noted with repeated activity and cold.12 Confirmation and AD MC in other families. Sequencing of the whole gene is of the diagnosis is made by establishing a pathogenic mutation needed, as one of the pitfalls for genetic testing is the presence in the SCN4A gene (www.genetests.org). Patients should receive 29 NONDYSTROPHIC MYOTONIC DISORDERS counseling on the effect of exercise, repeated activity, and 5. Miller TM. Differential diagnosis of myotonic disorders. AANEM cold as well as genetic counseling. Mexiletine can be used for monograph #27. Muscle Nerve 2008;37:293-299. symptomatic treatment. 6. Koch MC, Steinmeyer K, Lorenz C, Ricker K, Wolf F, Otto M, Zoll B, Lehmann-Horn F, Grzeschik K-H, Jentsch TJ. The skeletal Sodium Channel Myotonia muscle chloride channel in dominant and recessive human myotonia. Science 1992; 257:797-800. Sodium channel myotonia is caused by a mutation in the SCN4A 7. Thomsen, J. Tonische Kraempfe in willkuerlich beweglichen gene on chromosome 17q23, which encodes the alpha subunit Muskeln in Folge von ererbter psychischer Disposition: Ataxia of the voltage-gated sodium channel. The disorder is allelic muscularis? Arch Psychiat Nervenkr 1876;6:702-718. with PC, hyperkalemic periodic paralysis, and hypokalemic 8. Becker PE. Zur Genetik der Myotonien. In: Kuhn E. Progressive periodic paralysis type 2. It encompasses a group of various muskeldystrophie, myotonie, myasthenie. Berlin: Springer-Verlag; disorders, including myotonia fluctuans, myotonia permanens, 1966. pp. 247-255 and acetazolamide-responsive myotonia.20-23 All share the 9. Becker PE. Myotonia congenita and syndromes associated with tendency to worsen with potassium ingestion and lack of cold myotonia. Vol. III. Topics in human genetics. Stuttgart: Georg sensitivity. In addition, other characteristics that help differentiate Thieme; 1977. the group from MC patients are a delayed onset myotonia after 10. Trip J, Drost G, Ginjaar HB, Nieman FH, van der Kooi AJ, de Visser exercise (10-30 min) and a more frequent occurrence of eye M, van Engelen BG, Faber CG. Redefining the clinical phenotypes closure myotonia and muscle pain.3,10 Furthermore, transient of non-dystrophic myotonic syndromes. J Neurol Neurosurg weakness at initiation of movement reported with MC and cold- Psychiatry 2009;80(6):647-652. induced weakness reported with PC are not seen.3 Similar to other 11. Fournier E, Arzel M, Sternberg D, Vicart S, Laforet P, Eymard B, nondystrophic myotonia types, the clinical features are helpful in Willer JC,Tabti N, Fontaine B. Electromyography guides toward differentiating the subtypes; however, there is overlap, making subgroups of mutations in muscle channelopathies. Ann Neurol. genetic confirmation necessary. 2004;56(5):650-661. 12. Fournier E, Viala K, Gervais H, Sternberg D, Arzel-Hézode M, Serum CK is normal to mildly elevated. Needle EMG showed Laforêt P, Eymard B, Tabti N, Willer JC, Vial C, Fontaine B. diffuse myotonic discharges. The short exercise test shows a Cold extends electromyography distinction between ion channel normal response, indicative of pattern III. Patients should receive mutations causing myotonia. Ann Neurol 2006;60(3):356-365. genetic counseling and advice to avoid food rich in potassium. 13. Tan SV, Matthews E, Barber M, Burge JA, Rajakulendran S, Fialho Mexiletine can be used for symptomatic treatment. Acetazolamide D, Sud R,Haworth A, Koltzenburg M, Hanna MG. Refined exercise is helpful in some patients. testing can aid DNA-based diagnosis in muscle channelopathies. Ann Neurol 2011;69(2):328-340. CASE PRESENTATION FOLLOWUP 14. Raja Rayan DL, Haworth A, Sud R, Matthews E, Fialho D, Burge J, Portaro S, Schorge S, Tuin K, Lunt P, McEntagart M, Toscano A, The patient short exercise test showed a progressive CMAP Davis MB, Hanna MG. A new explanation for recessive myotonia decline with repeated trials. Genetic testing confirmed a sodium congenita: Exon deletions and duplications in CLCN1. Neurology. gene mutation SCN4 (theronine > methionine; codon 1313, 2012;78(24):1953-1958. nucleotide 3938) establishing the diagnosis of PC in the patient. 15. Trip J, Drost G, van Engelen BG, Faber CG. Drug treatment for The patient received mexiletine; however, she had difficulty myotonia. Cochrane Database Syst Rev. 2006. tolerating the medicine because of dyspepsia and dizziness. 16. Statland J, Salajegheh M, Bundy B, Wang Y, et al. Phase II In retrospect, the patient’s old records indicate a diagnosis of possible Therapeutic trial of mexiletine in non-dystrophic myotonia: neuromyotonia based on needle EMG findings. This is likely secondary outcomes show improvement in symptoms and signs of explained by the recently described, distinctive neuromyotonia- myotonia. Platform presentation (S55. 005). 65th Annual Meeting like discharges noted in some of the patients with PC. of the American Academy of Neurology. New Orleans, Louisiana. April 2012. REFERENCES 17. Ptacek LJ, Trimmer JS, Agnew WS, Roberts JW, Petajan JH, Leppert M. Paramyotonia congenita and hyperkalemic periodic 1. Griggs RC, Mednell JR, Miller RG. Periodic paralysis and paralysis map to the same sodium-channel gene locus. Am J Hum myotonia. Evaluation and treatment of myopathies. Philadelphia: Genet 1991;49: 851-854. F.A. Davis;1995, pp 26-29, 318-354. 18. Miller TM, Dias da Silva MR, Miller HA, Kwiecinski H, Mendell 2. Amato A, Russel J. Nondystrophic myotonias and periodic JR, Tawil R,McManis P, Griggs RC, Angelini C, Servidei S, paralysis. Neuromuscular Disorders, 1st ed. New York: McGraw- Petajan J, Dalakas MC, Ranum LP, Fu YH, Ptácek LJ. Correlating Hill; 2008, pp 655-680. phenotype and genotype in the periodic paralyses. Neurology 3. Matthews E, Fialho D, Tan SV, Venance SL, Cannon SC, Sternberg 2004;63(9):1647-1655. D, Fontaine B, Amato AA, Barohn RJ, Griggs RC, Hanna MG; 19. Eulenburg, A. Ueber eine familiare, durch 6 generationen CINCH Investigators. The non-dystrophic myotonias: molecular verfolgbare Form congenitaler Paramyotonie. Neurologisches pathogenesis, diagnosis and treatment. Brain 2010;133:9-22. Centralblatt 1886;12:265-272. 4. Fialho D, Schorge S, Pucovska U, Davies NP, Labrum R, Haworth 20. Trudell RG, Kaiser KK, Griggs RC. Acetazolamide responsive A, Stanley E,Sud R, Wakeling W, Davis MB, Kullmann DM, Hanna myotonia congenita. Neurology 1987;37:488-491. MG. Chloride channel myotonia:exon 8 hot-spot for dominant- negative interactions. Brain 2007;130:3265-3274. 30 NEUROMUSCULAR UPDATE II

21. Ricker K, Moxley RT III, Heine R, Lehmann-Horn F. Myotonia fluctuans: a third type of muscle sodium channel disease. Arch. Neurol. 1994; 51:1095-1102. 22. Lerche H, Heine R, Pika U, George AL, Jr, Mitrovic N, Browatzki M, Weiss T, Rivet-Bastide M, Franke C, Lomonaco M, Ricker K, Lehmann-Horn F. Human sodium channel myotonia: slowed channel inactivation due to substitutions for a glycine within the III-IV linker. J. Physiol 1993;470:13-22. 23. Colding-Jorgensen E, Duno M, Vissing J. Autosomal dominant monosymptomatic myotonia permanens. Neurology 2006;67:153-155.

31 32 Neuromuscular Vignettes

Annabel K. Wang, MD Associate Professor of Neurology University of California, Irvine Orange, California

Gregory T Carter, MD, MS Medical Director, Muscular Dystrophy Association Regional Neuromuscular Center, Olympia, Washington

Andrew W. Tarulli, MD Assistant Professor, Neurology Harvard Medical School Boston, Massachusetts

VIGNETTE ONE 1B. What other information would be helpful?

A 34-year-old man was evaluated for pain in feet. He described A. History of diabetes. the sensation as if a hot towel were wrapped around his feet. There was no weakness. Review of systems revealed dry eyes, blurred B. Ethnic background. vision, and lightheadedness. He was evaluated for intermittent abdominal pain and vomiting without nausea. He has also had C. Family history. episodes of erectile dysfunction. D. All of the above. His cranial nerve examination was normal. He has normal tone, bulk, and strength. Sensory examination revealed normal light 1C. What other studies would be MOST useful? touch, pinprick, vibration, and joint position sense. Reflexes were absent in ankles only. Cerebellar examination was normal. A. Genetic testing.

Questions B. Cardiac echocardiogram.

1A. What is the clinical pattern of his symptoms and signs? C. Autonomic testing.

A. Distal sensory loss. D. Serum immunofixation.

B. Sensory with upper motor neuron signs. VIGNETTE TWO C. Severe propriceptive deficit. A 43-year-old man, with a history of papillary thyroid cancer, devel- D. Sensory and autonomic. oped right arm abduction weakness after surgical resection. He was referred to the EMG Laboratory for evaluation. He has weakness in right abduction only. Sensory examination and reflexes were normal.

33 NEUROMUSCULAR VIGNETTES Questions 2C. What are the consequences of injury to the spinal accessory nerve?

2A. What is the cause of his weakness? A. Lateral and upward displacement of the scapula.

A. Spinal accessory neuropathy. B. Drooping of the shoulder.

B. Axillary neuropathy. C. Asymmetric shoulder shrug.

C. Brachial . D. All of the above.

D. Cervical . Below are the results of the patient’s electrodiagnostic (EDX) studies.

Routine nerve conduction studies (NCSs) and needle electromyography (EMG) of the right arm is normal.

2B. What other studies should be performed?

A. Spinal accessory nerve motor study.

B. Needle examination of the trapezius.

C. Needle examination of the sternocleidomastoid.

D. All of the above.

Motor NCSs

Nerve and Lat Amp Dur Area Temp Lat diff Dist CV site (ms) (mV) (ms) (mVms) (°C) (ms) (mm) (m/s)

Median R to abductor pollicis brevis R Wrist 3.5 12.7 6.2 48.3 33.5 3.5 70 Elbow 8.2 12.3 6.3 46.6 33.3 4.7 240 51

Ulnar R to abductor digiti minimi (manus) R Wrist 3 8.4 7.8 27.1 33.3 3 70

Below elbow 7 7.6 7 23.6 33.7 4 235 59

Above elbow 9.5 7.2 7.1 23.6 33.8 2.5 160 64

Accessory (spinal) R to trapezius R Neck 2.5 3.1 14.8 26 33.2 2.5

Accessory (spinal) L to trapezius L Neck 2.2 7.3 15.2 72.2 33.1 2.2

F-Wave studies

M-Lat F-Lat Nerve (ms) (ms)

Median R 3.7 29.2

Ulnar R 3.1 30.6

34 PRINCIPLES OF NCS AND NEEDLE EMG Sensory NCSs

Nerve and Onset lat Peak lat Amp Temp Lat diff Dist CV site (ms) (ms) (μV) (°C) (ms) (mm) (m/s)

Median R to digit II (index finger) R Wrist 2.3 3.1 33 33.9 2.3 130 57

Ulnar R to digit V (little finger) R Wrist 1.9 2.7 23 34 1.9 110 59

Radial R to anatomical snuff box R Forearm 1.8 2.3 25 33.9 1.8 100 56

Lateral antebrachial cutaneous R to forearm R Elbow 2.2 2.8 18 33.8 2.2 130 59

Medial antebrachial cutaneous R to forearm R Elbow 2.3 2.8 10 33.8 2.3 130 57

Lateral antebrachial cutaneous.L to Forearm.L Elbow 2.3 2.8 15 33.3 2.3 130 57

Medial antebrachial cutaneous.L to Forearm.L Elbow 2.5 2.9 9 33.3 2.5 130 52

Needle EMG Examination

Muscle Insertion Spontaneous activity Volitional MUAPs activity Fibs/PSW Fasc Other Amp Dur Poly Recruit Rate Comment Deltoid R Normal None None Normal Normal None Normal Normal Biceps brachii Normal None None Normal Normal None Normal Normal R Triceps brachii Normal None None Normal Normal None Normal Normal R Pronator teres Normal None None Normal Normal None Normal Normal R

First dorsal Normal None None Normal Normal None Normal Normal interosseous R

Abductor pollicis brevis Normal None None Normal Normal None Normal Normal R

Trapezius R Normal 2+ None Normal 2+ Many Mark Decr Normal Sat Portentials

Sternocleidom Normal 2+ None Normal 2+ Many Mod Decr Normal Sat Portentials astoid R Infraspinatus Normal None None Normal Normal None Normal Normal R C4 paraspinal Normal None None R

35 NEUROMUSCULAR VIGNETTES VIGNETTE THREE VIGNETTE FOUR

A 66-year-old man was evaluated for progressive walking A 53-year-old man had insidious difficulty over several months. He noticed the onset of tingling in onset of difficulty speaking his toes. He felt like he was walking as if he was drunk. He had and swallowing. He complains intermittent falls. of a hoarse voice, along with loss of muscle mass in all his His examination revealed normal tone, bulk, and strength. limbs. He was originally told by Vibration was absent in toes and joint position sense was mildly another neurologist (who was reduced in toes. His reflexes were 3+ in the upper extremities and not a member of the American 2+ in the lower extremities. His toes were upgoing. Association of Neuromuscular and Electrodiagnostic Medicine) Questions that he had amyotrophic lateral sclerosis (ALS) and had “only a 3A. What is the clinical pattern of his symptoms and signs? few years to live.” He is referred to you for a second opinion and A. Distal sensory loss. some management issues.

B. Sensory with upper motor neuron signs. Physical examination showed CERVICAL COLLARS diffuse muscle wasting and IN NEUROMUSCULAR C. Severe proprioceptive deficit. multiple fasciculations. You DISEASE also note hyporeflexia and what Sensory and autonomic. appears to be an intention tremor. Individuals with neck extensor D. muscle weakness may complain There was also diminished fine of neck stiffness, heaviness, and 3B. Which conditions lead to subacute combined degeneration? motor movement, and a “floppy fatigue in holding the head up, and head.” they may also notice difficulties Nitrous oxide abuse. in keeping the head upright with A. unexpected movements. In the Questions later stages, and in cases of severe B. Vitamin B12 deficiency. muscle weakness, the head drops 3A. Which additional finding is forward, the cervical spine is Copper deficiency. he also MOST LIKELY to have? completely flexed, and the patient C. experiences severe neck pain and anterior muscle tightness. D. All of the above. A. Ashworth grade II spastic- ity. Several different types of collars Laboratory and examination results showed the following: that can support the head, protect the weakened muscles, and prevent • EDX studies: Absent bilateral sural responses only. B. Cogwheel rigidity in his further deformity are available. • C-spine magnetic resonance imaging: Mild degenerative changes. extremities. For mild-to-moderate weakness • Vitamin B12, homocysteine, and methylmalonic acid levels: a soft foam collar is usually Normal. Gynecomastia. recommended. Soft collars are C. usually comfortable and well- • Copper: 62 (normal 85-155) mcg/dL. tolerated, and neck movements are • Ceruloplasmin: 30 (normal 22-58) mg/dL. D. Oculomasticatory myor- limited to a certain degree. • Zinc: 120 (normal 50-155) mcg/dL. hythmia. 3C. What other history would be helpful? For moderate-to-severe weakness, a semi-rigid collar, such as the Hyperhydrosis and allodynia traditional Philadelphia™ collar, A. Gastric surgery. is required. Patients find these The patient is not able to elevate collars very warm, may experience Excessive zinc ingestion. the soft palate and has markedly discomfort at points of contact and B. pressure over the trachea, or may atrophy and decreased tongue feel confined. If the individual has a C. Malabsorptive enteropathies. movement. Having recently tracheostomy, a Miami-J®, Aspen, passed the American Board of or Malibu collar that allows for All of the above. Electrodiagnostic Medicine ex- anterior neck access are prescribed. D. Warmth, discomfort, and a sense of amination, you are confident confinement are also common with in your needle EMG skills and these collars. The Headmaster, decide to restudy this man. Executive, and Canadian collars Needle EMG showed acute and have an open air design, but do not control as well for lateral instability chronic denervation with fas- as the Miami-J®, Aspen, or Malibu ciculations; peripheral NCSs collars. In practice, the Headmaster showed decreased amplitude of collar is well accepted by many compound motor action poten- patients and is a compromise tials but no conduction slowing 36 NEUROMUSCULAR UPDATE II

between providing maximal or block. However, you had D. Scoliosis with a curve that likely exceeds 50 degrees and stability and comfort. great difficulty obtaining any forced vital capacity (FVC) is < 25% predicted value, sensory nerve action potentials. which is not corrected with spinal instrumentation. Patients with combined cervical and upper thoracic weakness may benefit from a cervical- 4B. What is his LIKELY diagnosis? 5B. Knowing that he has a serum creatine kinase level of 20,000 U/L thoracic orthosis, or a sterno- at rest, what is the BEST next test to perform in order to arrive at a occipital mandibular immobilizer A.Amyotorphic lateral sclerosis. definitive diagnosis? (SOMI). These devices provide maximum support, but they are more expensive, heavy, and may B.Multi-focal motor neuropathy. A. EDX evaluation including single fiber EMG. be difficult to don and doff. For severe or intractable cervical or C.Spinal muscular atrophy B. Forearm ischemic exercise testing. trunk weakness, referral to an orthotist or biomedical engineer III (Kugelberg-Welander for a custom-made device may be syndrome). C. DNA evaluation. necessary. D.Spinobulbar muscular D. Muscle biopsy through needle aspiration. atrophy (Kennedy disease). 5C. One year later spinal radiographs reveal scoliosis with a E. Guillain-Barré syndrome primary curve exceeding 25 degrees and FVC is < 50% predicted value. What is the next management course? 4C. The patient has tremendous difficulty holding his head up and really wants help with this. What type of collar might work BEST in A. Aggressive spinal manipulation to reduce the curvature. this situation? B. Referral to a pediatric spine surgeon for spinal fusion and A. Freeman or Headmaster type cervical orthosis. instrumentation.

B. Minerva brace. C. Fitting with a custom molded spinal orthosis.

C. Halo type orthosis. D. Proprioceptive neuromuscular stretching through a neurode- velopmental therapist. D. The “Iron Maiden” neck stabilizer. 5D. When is spinal instrumentation in boys like this indicated? VIGNETTE FIVE A. Any time before the age of skeletal maturity. A 9-year-old boy presents to the neuromuscular clinic for evalu- ation and management. The family is from Mexico and the B. When the primary curve exceeds 25 degrees and FVC is < parents do not speak English. The boy speaks limited English. 50% predicted value. You are able to ascertain through an interpreter that he had an older brother who died of pneumonia when he was 18 years C. When the primary curve is less than 25 degrees. old. He also had a maternal uncle who died “very young” from some type of heart problem. The boy is now having progressive D. Never because scoliosis is not corrected with spinal instru- difficulties walking and is falling all the time at school now. On mentation. examination you note reflexes are 1+ in the arms and absent in the legs, with pinprick and vibratory sensation intact. The boy E. Never because scoliosis is best corrected with bracing. has a distinct waddle when he walks and is up on his toes. VIGNETTE SIX Questions A 28-year-old man presents with a 5-7 year history of slowly 5A. Based on the photograph above, what additional findings does progressing weakness beginning in the distal limb muscles but he also have? now affecting his hands. He is starting to notice muscle wasting. He also remembers that as a child he was always clumsy or not A. Hyperlorodotic positioning of the spine with heel cord very athletic. tightness. He now notes that he is having great difficulty walking and he B. Severe pectus excavatum. frequently trips due to catching his feet. Frequent ankle sprains and falls are also a problem for him. C. Kyphosis with a secondary curve less than 20 degrees. On examination, his reflexes are absent throughout. He demon- strates 4/5 strength in most major muscle groups. After walking for several minutes he does start to drag his feet.

37 NEUROMUSCULAR VIGNETTES Questions VIGNETTE SEVEN

6A. Which type of bracing modality is MOST LIKELY to improve his gait?

A. Custom fit shoe orthotics with metatarsal pads.

B. Bilateral double metal upright ankle-foot orthotics (AFOs).

C. Lightweight, low profile AFOs with dorsiflexion assist.

D. Bilateral long leg, double metal upright knee ankle-foot orthotics (KAFOs).

E. Bilateral universal foot orthotics (UFOs).

6B. Which factor is MOST LIKELY to contribute to loss of ambula- tion in a patient with ?

A. Progressive dyspnea. A 57-year-old woman is referred to your office with slowly pro- gressive weakness in both proximal and distal muscles. She is B. Weakness in the quadriceps muscles (< 3/5 on manual much weaker on the left side of her body with striking asymme- muscle testing). try. She has remarkable weakness of the wrist and finger flexors, disproportionate to that of her extensors (see image). Hence, she C. Severe atrophy in the cerebellum. has loss of finger dexterity and grip strength.

D. Loss of pain sensation. Questions

E. Weakness (< 4/5) in tibialis anterior muscles. 7A. All of the following are likely additional symptoms, EXCEPT:

6C. Which gait description BEST characterizes his walking? A. Dysphagia.

A. Toe walking. B. “Floppy neck” syndrome.

B. A steppage gait, where the patient lifts their legs in an ex- C. and cramping. aggerated fashion to clear the feet off the ground. D. Sensory and autonomic dysfunction. C. A reverse Trendelenberg gait caused by gluteus medius weakness. DEVICES TO ASSIST WITH WALKING: MOBILITY AIDS D. A shuffling gait due to impaired proprioception. Muscle weakness and resultant balance problems will eventually necessitate that a patient use an aid to enhance mobility and safety. The type of assistive device E. A VIP gait often noted in executives and politicians. recommended is determined by the degree of lower extremity (LE) and trunk weakness and spasticity, range of motion and strength of the upper extremities 6D. Regarding orthotics in peripheral neuropathy, which statement is (UEs), extent and rate of progression of the disease, acceptance of the aid by the MOST appropriate and correct? the individual, and economic constraints. In addition to taking into account which device will ensure optimal function and safety, the weight of the mobility aid is an important factor to consider in decisionmaking, as individuals with A. Custom fit shoe orthotics with metatarsal pads are most amyotrophic lateral sclerosis (ALS) experience muscle fatigue. helpful in terms of improving balance. Canes

B. Bilateral KAFOs are generally too heavy and cumbersome Canes are usually recommended in the early stages of ALS for mild LE weakness to be of much use. or balance problems, as they provide the least amount of support. A cane is carried in the hand opposite to the most affected leg, requires good UE strength, UFOs cannot be used outside of Area 51. and can be used on stairs. The standard wooden cane is the least expensive, has a C. curved handle, and can be made shorter but not longer. An adjustable aluminum cane easily adapts for various heights and is lightweight. An aluminum cane D. Bilateral double metal upright AFOs are often preferred may have a curved handle or an offset handle. An offset handle allows the by patients as they provide excellent medial-lateral ankle weight to be directed over the cane tip when in contact with the floor, rather support and stability. than anteriorly, as is the case with a curved handle. Quad canes are aluminum canes with four points of floor contact. Quad canes provide greater stability than straight canes, but all tips must be in contact with the ground for stability. The E. Upper extremity orthotics are easy to use and very helpful size of base can vary and these canes are heavier to lift. Aluminum and quad 38 for patients with peripheral neuropathy. NEUROMUSCULAR UPDATE II

canes come in a variety of styles 7B. She continues to have slow but ill-defined hyperdensity overlying the gastric bubble. Nerve con- and sizes of handgrips. Patients relentless progression of muscle duction studies show symmetrically reduced peroneal and tibial with hand muscle weakness may weakness and is now having diffi- CMAP amplitudes with normal conduction velocities, reduced be better able to grip an enlarged or molded handle. culty with ambulation and frequent sural SNAP amplitudes with normal conduction velocities, and falls. What one examionation/test normal NCSs in the arms. Needle EMG shows fibrillation poten- Crutches should you be sure to perform? tials and reduced recruitment in tibialis anterior, extensor hallucis longus, and gastrocnemius, but is otherwise normal. In general, crutches are rarely recommended for individuals A. Bone density scan to assess with ALS, as the patient must for osteoporosis. Questions have very good UE and trunk strength as well as adequate Hemoglobin A1c blood 8A. Which of the following is the MOST LIKELY diagnosis? balance to use them. If crutches B. are recommended, Loftstrand or test to rule out diabetes. Canadian crutches are preferred. A. Acute intermittent porphyria. These crutches consist of a single C. Opthamology examination upright, a forearm cuff, and a to assess for cataracts. Arsenic intoxication. handgrip; the hands can be freed B. for standing tasks without having to release the crutch. D. Hepatobiliary iminodiace- C. Guillain-Barré syndrome. tic acid scan to assess for Walkers gall bladder sludging. D. Fabry’s disease. Walkers provide greater support than canes and crutches, but 7C. What is the MOST LIKELY E. Polyarteritis nodosa are more bulky and may be cause of her difficulty in walking? cumbersome in confined spaces. 8B. Which of the following statements concerning this patient is Various types of walkers are available (e.g., folding, with or A. Weakness in her spinal ex- TRUE? without wheels, with brakes, with tensor musculature. seating surfaces, etc.) and some A. She is likely to have diffuse hair loss. walkers can be modified to suit the Loss of proprioception. individual’s walking environment B. and needs; for example, they B. Her transaminases should be normal. may be fitted with a basket, food C. Weakness in her tray, or forearm trough. Standard quadriceps muscles. C. Mees’ lines (leukonychia) are a frequent finding. aluminum walkers are the least expensive, very stable, and can be adjusted for various heights. D.Loss of full field of vision. D. Motor symptoms are more likely to precede sensory symptoms. However, they must be picked up and lowered during ambulation, 7D. You want to prescribe some- E. Recent fish consumption may result in an increase in the and patients may find them heavy thing to assist her in walking. What measured arsenic level. to lift. Walkers with wheels do not need to be lifted and they would be BEST to reduce her risk roll forward easily, thus they are of falling? 8C. Which of the following is the MOST appropriate for this usually recommended for patients patient? with ALS; however, the stability Single point cane. of wheeled walkers is decreased A. and they may move forward too A. Chelation with dimercaprol. quickly. Some walkers come B. Front wheeled walker. equipped with brakes. Push down B. Gastrointestinal lavage. brakes secure a walker when the Quad (four-point) cane. patient loads his or her weight on C. the walker and are preferred over C. Hematin and glucose. squeeze type brakes for patients D. A hemi walker. with hand weakness. Specialized D. Intravenous immunoglobulin. wheeled walkers are the most expensive. They have large wheels VIGNETTE EIGHT that can move over a variety E. Prednisone. of terrains. A 40-year-old woman presents with 2 weeks of burning and VIGNETTE NINE tingling pain in her fingers and toes followed by progressive leg weakness. She also has a 2-year history of repetitive vomiting and An 18-month-old girl (see photo) presents for evaluation of weak- diffuse abdominal pain. On examination, she has severe distal ness. She has a previous history of neonatal hypotonia, with re- lower extremity weakness with bilateral foot drop, distal sensory spiratory failure requiring intubation and developmental delay. On loss in a stocking-glove pattern, and absent deep tendon reflexes. physical examination, she has a long face, bilateral facial weakness, Her skin is diffusely hyperpigmented. Laboratory studies are sig- open jaw, bilateral ptosis, and moderate distal-greater-than-proximal nificant for a hematocrit of 28% with a mean corpuscular volume muscle weakness. Hand-grip and percussion myotonia are present. of 85 fL. Basophilic stippling is present. Chest X-ray shows an

39 NEUROMUSCULAR VIGNETTES Questions and left external shoulder rotation which is 5−/5. There were no reflex asymmetries. Sensation was minimally decreased over the 9A. What is the MOST LIKELY diagnosis? lateral left shoulder. Needle EMG and NCSs show the following:

A. Facioscapulohumeral dystrophy. Sensory NCSs

Recording Onset Peak NP amp Temp Dist Vel B. Myotonic dystrophy type 1. Nerve/sites site (ms) (ms) (μV) (°C) (cm) (m/s) C. Myotonic dystrophy type 2. Left median 1. Wrist Digit II 2.7 3.5 20.1 14 51.9 D. Myotonia congenita. Left ulnar 1. Wrist Digit V 2.35 3.65 14.3 32.4 12.4 52.8 E. Slow-channel syndrome. NP amp = negative peak amplitude, Dist = distance, Temp = temperature, 9B. How did this patient MOST Vel = velocity LIKELY inherit this disorder? Motor NCSs A. Autosomal dominant, from Recording Lat Amp Area Temp Dist Vel her father. Nerve/sites Site (ms) (mV) (mVms) (°C) (cm) (m/s) B. Autosomal dominant, from Left median, APB her mother. 1. Wrist APB 3.65 11.6 43.4 32 7 2. Elbow APB 7.95 11.6 41 22.4 52.1 C. Autosomal recessive, from Left ulnar, ADM her father. 1. Wrist APB 2.7 14.9 45.9 7 2. Below APB 6.7 13.9 45.5 22.9 57.2 D. Autosomal recessive, from elbow 3. Above APB 7.85 12.4 41.9 7.2 62.6 her mother. elbow

E. It is X-linked recessive. ADM = abductor digiti minimi, Amp = amplitude, APB = abductor pollicis brevis, Dist = distance, Lat = latency, Temp = temperature, Vel = velocity 9C. What is the LIKELY number of CTG repeats in the DMPK gene in F Wave Studies this patient? Fmin Fmax Nerve A. < 30. (ms) (ms)

Left median, B. 50. 31.15 31.65 APB

Left ulnar, C. 100. 30.55 33.6 ADM D. 250. ADM = abductor digiti minimi, APB = abductor pollicis brevis E. 1,000. Needle EMG Summary Table

VIGNETTE TEN Spontaneous MUAP

Insertional Fib Fasc Comment Dur Amp Phases Activation Recruitment A 63-year-old man presents with left CRDS, Slightly Left deltoid Increased 2+ None 2+ 2+ 1+ Normal shoulder pain and atrophy. Two years PSWs reduced ago, while lifting weights, he noticed Left biceps Normal None None - N N N Normal Normal the sudden onset of left shoulder pain Left triceps Normal None None - N N N Normal Normal Left teres without radiation. The pain did not Normal 1+ None PSWs 1+ 1+ 1+ Normal Normal minor improve on its own, so he began phys- Left Normal None None - N N N Normal Normal ical therapy. After 20 sessions, his pain infraspinatus had improved, but he also noticed that Left his left deltoid was becoming atro- rhomboid Normal None None - N N N Normal Normal phic. Directed neurological examina- major tion shows atrophy of the left deltoid. Left C5 PSP Normal None None - N N N Normal Normal Strength is normal with the exception of left shoulder abduction which is 4/5 Amp = amplitude, CRD = complex repetitive discharges, Dur = duration, Fib = 40 fibrillation, Fasc = fasciculation, PSP = paraspinal, PSW = positive sharp wave NEUROMUSCULAR UPDATE II Questions When the patient’s medical records arrive, you learn that he had been given bortezomib to treat his multiple myeloma. 10A. What is the MOST LIKELY localization of this lesion? 11B. Which of the following is TRUE about the development of poly- A. Axillary nerve. neuropathy secondary to this medication?

B. C5 nerve root. A. It is an acute-onset neuropathy characterized by paresthe- sias, allodynia, and cold hypersensitivity. C. Lateral cord of brachial plexus. B. Symptoms resolve immediately upon discontinuation of D. Musculocutaneous nerve. bortezomib.

E. Upper trunk of brachial plexus. C. The neuropathy usually presents within the first five cycles of administration. 10B. Which of the following sensory NCSs would help to distinguish this condition from an upper trunk brachial plexopathy? D. The neuropathy appears several months after completion of chemotherapy. A. Lateral antebrachial cutaneous nerve. E. It is difficult to distinguish from POEMS (, B. Medial antebrachial cutaneous nerve. organomegaly, endocrinopathy, monoclonal gammopa- thy, and skin changes) syndrome on electrophysiologic C. Median nerve, recording at digit III. grounds.

D. . 11C. Which of the following agents is effective in prophylaxis of bortezomib-induced neuropathy? E. None of the above. A. Calcium. 10C. What is the MOST LIKELY site of nerve entrapment in this case? B. Magnesium. A. Arcade of Frohse. C. Amitryptiline. B. Quadrilateral space. D. Alpha lipoic acid. C. Spinoglenoid notch. E. None of the above. D. Suprascapular notch.

E. Thoracic outlet.

VIGNETTE ELEVEN

A 78-year-old man with multiple myeloma undergoing chemo- therapy presents with pain and paresthesias in the hands and feet. Unfortunately, he does not know which chemotherapeutic agent he has received and no medical records are available for review.

Questions

11A. Which of the following chemotherapeutic agents used for mul- tiple myeloma is MOST LIKELY to have produced his symptoms?

A. Bortezomib.

B. Dexamethasone.

C. Lenalidomide.

D. Melphalan.

E. Neuropathy is a common side effect of all the agents listed. 41